Location via proxy:   [ UP ]  
[Report a bug]   [Manage cookies]                

Osmosis Gastro

Download as pdf or txt
Download as pdf or txt
You are on page 1of 43

1: Trastornos esofágicos

A 47-year-old man comes to the clinic because of food sticking to his throat, a sensation
that has grown gradually worse over the last 5-7 years. He has dysphagia for solid food
and liquids, and often spits food back up undigested, along with a large amount of saliva.
He reports severe discomfort after a large meal and has lost 6.8-kg (15-lb) over the past
three years unintentionally. He says he has substernal chest pain and cannot remember the
last time he vomited, although he does frequently hiccup. He does not want to lose
additional weight and wants to understand the situation and what treatment options are
available. Which of the following tests is most appropriate to confirm the probable
diagnosis? Manometry

A 40-year-old woman comes to the clinic because of bilateral, symmetrical swelling of her
hands and fingers, as well as dysphagia with both solids and liquids. Physical examination
shows telangiectasis on her forehead (see picture), and her phalanges are positive for
Raynaud's phenomenon. Which of the following antibodies are most likely to be found on
laboratory testing? Anti centromere

A 32-year-old man comes to the clinic because of a progressive history of painful


swallowing of solids and liquids. He has putrid breath and esophageal manometry shows
an elevated lower esophageal sphincter pressure. A barium swallow study is obtained and
the results of which are shown below. Which of the following is the most likely diagnosis?
Achalasia

A 72-year-old man comes into the outpatient clinic because of difficulty swallowing. For
several years he had difficulty swallowing solids, but recently developed problems
swallowing liquids. He denies fever, chills, nausea, vomiting, and unintentional weight loss.
The patient does not smoke cigarettes and does not drink alcohol. His temperature is
37.0°C (98.6°F), pulse is 80/min, respirations are 15/min, and blood pressure is 125/85 mm
Hg. Barium swallow study with fluoroscopy shows a diverticulum arising from the midline
of the posterior wall of the distal pharynx. Which of the following is the most appropriate
next step in management? Cricopharyngeal myotomy and diverticulectomy

A 52-year-old woman comes to the urgent care clinic because of progressive dysphagia for
solids and liquids. She has putrid breath and esophageal manometry shows elevated lower
esophageal sphincter pressure. A barium swallow study is obtained and the results of
which are shown below. Which of the following organisms is most likely to be implicated?
Trypanosoma cruzi

A 49 year-old woman comes to the clinic because of tightening of the skin on her hands
and face. She has a history of hypertension and asthma. She is currently taking amlodipine,
losartan, and salbutamol. She does not smoke or drink alcohol. Her temperature is 37.5°C
(99.5°F), pulse is 70/min, respirations are 14/min, and blood pressure is 145/87mmHg.
Physical examination shows non-pitting edema and some ulceration on her hands. Her
abdomen is not distended. Bowel sounds are hypoactive. She does not have stridor but
there is some diffuse bilateral wheezing on auscultation with good air entry bilaterally.
Cardiac examination is normal. Which of the following organs is most likely to be affected
by the patients disease? Esophagus

A 58-year-old woman comes to the urgent care clinic because of a several month history of
progressive pain with swallowing. She has difficulties tolerating solids or liquids and also
has foul-smelling breath. She has no known allergies and has recently traveled to South
America. A barium swallow study is obtained and the results of which are shown below.
Which of the following is the most appropriate next step in management for this patient?
Injection of botox in the lower esophageal sphincter

A 58-year-old woman comes to the urgent care clinic because of progressive dysphagia for
solids and liquids over the last 6 months. She has putrid breath and esophageal
manometry shows elevated lower esophageal sphincter pressure. A barium swallow study
is obtained and the results of which are shown below. Which of the following treatment
options is the most appropriate management choice for this patient? Botulinum injection

A female neonate is delivered vaginally at full term to a 23-year-old woman who has not
had close prenatal follow-up throughout her pregnancy. Several hours after birth, the girl
begins to cough and subsequently experiences choking episodes. Examination shows that
the infant is cyanotic. Resuscitation efforts are initiated and her condition stabilizes. Which
of the following is the most likely diagnosis? type c esophageal atresia with distal
tracheoesophageal fistula

A 55-year-old woman is brought to the emergency department with acute onset chest pain
for the past 30 minutes. The pain started suddenly during dinner. The patient describes the
pain as crushing substernal pain and rates it as 8/10. She recalls a few occasions of similar
pain episodes but did not seek treatment because she thought it would pass. Medical
history is significant for smoking 1 pack per day for 30 years. The patient has not noticed
any relation to specific foods and does not have any reflux symptoms. The patient took her
partner’s sublingual nitroglycerin pill before the ambulance arrival, which provided partial
relief. Temperature is 37.0°C (98.6°F), pulse is 90/min, and blood pressure is 135/85 mmHg.
ECG is performed and shows normal sinus rhythm with no significant ST-changes. A
barium swallow is performed and is shown below: Which of the following is the best initial
treatment for this patient's condition? Peppermint oil
A 43-year-old man comes to the clinic because of a 7-month history of progressive pain
upon swallowing. He has trouble tolerating solids and liquids and has developed
increasingly foul breath. He often regurgitates undigested food and has had a chronic
cough. A barium swallow is obtained and is shown below. Which of the following types of
peristalsis pattern is characterized by this patient's condition? Tertiary peristalsis

A 2800-g (6.17-lb) female infant is born vaginally and at term to a 21-year-old woman
who failed to have close prenatal follow-ups throughout her pregnancy. Shortly after birth,
the infant begins to cough and subsequently undergoes choking episodes. She is cyanotic.
Resuscitation efforts are initiated and her condition stabilizes. The following day
time-lapsed images of barium swallow are taken and are shown below. Which of the
following is the most likely diagnosis? Congenital tracheoesophageal fistula

A 28-year-old man comes to the emergency department because of vomiting, sudden


onset severe retrosternal chest pain, and strange popping sensations under his fingers
when he pushes on the skin in his upper chest. He was well until yesterday evening when
he developed nausea and vomiting several hours after a fast-food meal. Physical
examination shows subcutaneous emphysema. Which of the following is the most likely
diagnosis? Boerhaave syndrome

A 33 year-old man comes to the gastroenterologist because of progressive dysphagia over


the last 3 months. Barium swallow shows a dilated esophagus with an area of distal
stenosis. Upon surgical treatment of the patient's condition, which of the following is the
patient at the greatest risk of developing? Gastro-esophageal reflux disease

A 55-year-old man comes to the emergency department because of several episodes of


vomiting. The patient complains of retrosternal chest pain, back pain, and shortness of
breath. Auscultation shows diminished breath sounds bilaterally at the bases, and a
crackling sound heard best directly over the heart. Palpation of the anterior chest is best
described as feeling like there are "rice crispies" underneath the skin. Which of the
following tests would confirm the diagnosis? Gastrografin swallow study

A 62-year-old man who immigrated 10 years ago from Colombia is referred to a


gastroenterologist because of difficulty swallowing for the past 5 months. The patient
denies smoking, drinking, or pertinent family medical history. He does mention that lately
he has been experiencing shortness of breath and swelling of both of his legs. A barium
swallow study is completed and the results are shown below. Esophageal manometry
confirmed the diagnosis. What is the pathological agent behind a common cause for the
secondary form of this disease? Trypanosoma cruzi
A 62-year-old man who immigrated 10 years ago from Colombia is referred to a
gastroenterologist because of difficulty swallowing for the past 5 months. The patient
denies smoking, drinking, or pertinent family medical history. He does mention that lately
he has been experiencing shortness of breath and swelling of both of his legs. A barium
swallow study is completed and the results are shown below. Esophageal manometry
confirmed the diagnosis. What is the most likely underlying cause of this patient's
condition? Chagas disease

2: ERGE + Enfermedad + ácido péptica + cáncer gástrico


A 6-month-old boy comes to the office because of spitting up his meals for the past five
months. He is also often unsettled, with colicky behavior after feeding. Other than this, his
mother has no concerns. He is developing normally, and his growth has been stabilized at
the 90th percentile for his height and weight. Examination shows a well-appearing child.
Abdominal examination shows no abnormalities. Which of the following is the most likely
diagnosis? Gastroesophageal reflux disease

A 42-year-old woman comes to the office because of hematemesis and melena for the past
week. She has been experiencing abdominal pain, which she currently rates a 4 on a
10-point scale. She says that the pain worsens when she eats and that she often develops
nausea after eating. The pain is worse, particularly when eating fried foods. Physical
examination shows tenderness to palpation in the epigastric area, but no other
abnormalities. Which of the following is the most likely cause of her symptoms? Hiatal
hernia

A 60-year-old man comes to the emergency department because of sub-sternal chest pain
for an hour. He has experienced this pain before, typically after heavy meals, during times
of stress, and when he goes running in the park. He also feels the pain at night when he is
lying in bed and has previously been woken from sleep by discomfort. He has not noticed
any dyspnea, diaphoresis, or palpitations but is currently experiencing some nausea and a
sour taste in his mouth. Medical history is non-contributory. ECG shows normal sinus
rhythm. Which of the following is the most likely diagnosis? Gastroesophageal reflux

A 63-year-old man comes to the office because of heartburn for the past four months. He
experiences a burning feeling in his chest whenever he eats. He has also had difficulty
swallowing his food. The patient has lost 6.8-kg (15-lb) over the past four months. A chest
radiograph is obtained and shown below. Which of the following is the most likely
diagnosis? Sliding hiatal hernia

A 25-year-old man comes to the emergency department because of acute onset


hoarseness of voice within the past 3 days. He also reports coughing, frequent throat
clearing, and flu-like symptoms. He reports having an upper respiratory infection from
babysitting his younger cousin 5 days ago. A laryngoscopy is done and shows
erythematous and edematous laryngeal tissue. Which of the following is the next best step
in management for this patient? Drinking fluids and resting the voice

A 55-year-old man comes to the office because of severe diarrhea for the past three days.
He says his diarrhea developed suddenly, and there have been no associated symptoms.
Medical history includes an over-the-counter medication for heartburn, and regurgitation
for the past five days. He says these have been present for the past six months, and occur
daily but are not getting worse. Examination shows an afebrile man with a large body
habitus. Abdominal examination is non-contributory. A barium study is obtained. Which of
the following medications is the patient most likely taking for his heartburn? Magnesium
hydroxide antacid

A 5-month-old infant is brought to the office because of a cough and vomiting for a month.
The vomiting is worse following meals, and the infant is exclusively formula fed. The
mother says these symptoms have been progressively worsening. She says there has been
no projectile or bilious vomiting, fever, sputum production, or lethargy. She states she keeps
the child upright while feeding and has tried to switching formulas without improvement.
Physical examination shows the infant is in the 12th percentile for height and weight. The
remainder of the examination shows no abnormalities. Which of the following is the most
appropriate initial management? Formula thickening

A 55-year-old Caucasian man comes to the office because of progressively worsening


gastroesophageal reflux for the past nine months. His medical history includes severe
gastroesophageal reflux disease that was unresponsive to proton pump inhibitors. Nissan
fundoplication was performed a year ago and, initially controlled his symptoms adequately.
His current symptoms are worse than they were before the operation. Upper
gastrointestinal endoscopy shows high grade dysplasia of the distal esophagus. Which of
the following is the most appropriate next step in management? Esophagectomy

A 58-year-old woman comes to the office because of heartburn for two weeks. The pain
most often appears as she lies down to sleep. She also frequently wakes up with burning in
her throat and epigastric area. She sometimes has an unpleasant bile taste in her mouth.
She states that she has recently been having difficulty swallowing in addition to her usual
symptoms. She has hypertension and hypothyroidism. Her current medications include
Lopressor 50 mg daily, and Synthroid daily. Her temperature is 36.8°C (98°F), pulse is
87/min, respirations are 15/min, and blood pressure is 134/86 mm Hg. Which of the
following is the most appropriate next step? Endoscopy
A 65-year-old man comes to the office because of dull chest pain, dyspnea, and heart
palpitations. He occasionally feels his food "balling up" in his throat, causing significant
discomfort in his chest. He denies exacerbation of his chest discomfort during exercise. He
has tried taking calcium carbonate tablets to ease his symptoms with some success. His
chest X-ray is shown below. Which of the following is the most appropriate surgical
management of the patient's underlying condition? Nissen fundoplication

A 2-month-old infant is brought to the office because of vomiting and cough for a month.
The vomiting is worse following meals, and the infant is exclusively formula fed. The
mother states these symptoms have been worsening. She says there has been no projectile
or bilious vomiting, fever, sputum production, or lethargy. She says she keeps the child
upright while feeding and has tried to switching formulas without improvement. Physical
examination shows the infant is in the 5th percentile for height and weight. The remainder
of the examination shows no abnormalities. Which of the following is the most appropriate
diagnostic investigation? Esophageal pH monitoring

A 49-year-old man comes to the office because of difficulty swallowing for three months.
He says food has been getting "stuck" when he tries to swallow it. This sensation only
happens with food, especially bread, and not with liquids. He has tried many
over-the-counter remedies, including a medication which decreases stomach acid secretion.
These treatments have had little effect. His medical history includes persistent "heartburn"
for the past five years. A barium swallow is obtained. Which of the following is the most
likely diagnosis? Esophageal stricture

A 47-year-old Japanese man comes to the clinic because of a 6-month history of early
satiety and mild epigastric pain, as well as an unintentional weight loss of 6.8-kg (15-lb)
over the past year. He is examined by endoscopy, and biopsy shows gastric carcinoma. A
CT with contrast is obtained and shows an area that is concerning for metastasis to the
liver. Which of the following is the most appropriate treatment? Surgical resection of
tumors and adjacent lymph nodes

A 61-year-old man comes to the clinic because of dysphagia and “heartburn,” which he
states have become more troublesome over the past 5 months. Examination shows mild
tenderness to palpation over his epigastrium. Upper gastrointestinal barium swallow
fluoroscopy demonstrates a subdiaphragmatic gastroesophageal junction, but apparent
herniation of the gastric fundus superiorly and into the left hemithorax. Which of the
following is the most appropriate management of this patient’s underlying condition?
Surgical gastropexy
3: Sangrado de tubo digestivo
A 2-year-old girl is brought to the clinic by her mother because of skin spots. She says the
spots are patches of violaceous, compressible nodules on the girl’s lips, arms and chest as
shown in the image. The mother says that her husband has similar lesions and has been
hospitalized several times. In addition to the skin, which of the following is most commonly
affected? Small bowel

A 45-year-old man is brought to the emergency room after an episode of hematemesis. The
patient was watching television when he felt nauseated and produced bright-red vomitus.
Past medical history is notable for an ankle sprain, for which he is currently taking aspirin,
and Helicobacter pylori infection, which was successfully treated 7 months ago. The patient
consumes 4-6 beers per week and does not use tobacco products. His vitals are within
normal limits. Endoscopy is performed and examination of the stomach reveals a protruding
2.5-mm arteriole that has eroded the overlying mucosa. There is active spurting of blood
from the vessel. Which of the following is the next best step in the management of this
patient? Thermal coagulation of bleeding site

A 52-year-old man is brought to the emergency department after complaining of


excruciating chest pain that radiates to the back. He also reports shortness of breath and
pain with swallowing. The symptoms began 30 minutes ago, after the patient consumed a
large quantity of alcohol and had several episodes of forceful vomiting. Past medical history
is notable for hypertension, alcohol use disorder, and liver cirrhosis. His temperature is
37.2°C (99.0°F), blood pressure is 142/78 mmHg, and pulse is 75/min. Physical exam
shows crepitus on palpation of the upper chest wall. Abdominal exam is notable for
hepatosplenomegaly. Which of the following is the next best step in the management of
this patient? Perform contrast esophagram

A 65-year-old man comes to the emergency department for worsening abdominal pain and
intermittent black, tarry stools for the past three months. His medical history is significant
for chronic hepatitis B infection. Physical examination shows a frail gentleman with
decreased muscle mass, a protuberant abdomen, and icteric sclera. Abdominal ultrasound
shows enlarged lymph nodes in the hepatoduodenal ligament. Which of the following
structures is most likely to be compressed by these lymph nodes? Portal triad

A 70-year-old woman comes to the emergency department because of blood in her stool.
She says that her stool has been black and tarry, but she can occasionally see red streaking.
She reports no recent weight loss or abdominal pain. Her past medical history is significant
for Hepatitis C infection. Stool guaiac cards are positive. Endoscopy shows long, red streaky
areas on the stomach. Which of the following is the most likely diagnosis? Gastric antral
vascular ectasia
A 42-year-old woman who was hit by a car while riding her bike presents to the emergency
department with a pulse of 100 and a BP of 90/75. She is complaining of pain in the left
shoulder and wrist, with the latter showing an obvious deformity. A FAST scan is positive
for free fluid in the abdomen. Which of the following organs is most likely injured? Spleen

A 62-year-old man comes to the office because of painless rectal bleeding for the past 3
months. He describes the intermittent occurrence of streaks of “bright red blood” on the
toilet paper after wiping, and blood on but not mixed within the stool. Occasionally he has
noted a small volume of blood within the toilet bowl, and associates this with straining. For
the past two weeks he has noticed an “uncomfortable lump” in his anus when defecating,
which goes away by itself immediately afterwards. He says he has no abdominal pain,
weight loss, or fevers. He is a well-appearing man with obesity. Digital rectal examination
shows bright red blood on the examination glove following the procedure. Anoscopy shows
enlarged blood vessels above the pectinate line. Which of the following is the most
appropriate next-step in management? Rubber band ligation

A 55-year old woman comes to the emergency department because of light headedness,
fatigue, and a racing heart for 6 hours. She tells you that she has also had very dark stools
in the last day. Her medical history includes hyperlipidemia, and daily cigarette smoking.
Her temperature is 36.8°C (98°F), pulse is 110/min, respirations are 23/min, and blood
pressure is 90/65 mm Hg. Abdominal examination is noncontributory. A biopsy from upper
gastrointestinal endoscopy shows partial replacement of the gastric mucosal epithelium by
intestinal metaplasia in the antrum. Which of the following most likely explains this
patient’s problem? Curvilinear Gram-negative rod

4: Hepatopatías no infecciosas
A 55-year-old homeless woman comes to the emergency department because of acute
confusion. She is jaundiced, and her breath smells of alcohol. Shortly after arriving she falls
into a stupor and becomes unresponsive. She is stabilized and the following laboratory
values are obtained: Which of the following is the most likely cause of the patient's
increased PT? Increased fibrinolysis

A 55-year-old man comes to the emergency department because of increasing right upper
quadrant pain and jaundice. Physical examination shows mild scleral icterus, a 12-cm
(4.72-in) liver span and vague right upper quadrant pain on palpation. Laboratory studies
show: Which of the following is the most likely diagnosis? Alcoholic liver disease

A 35-year-old woman comes to the emergency department because of nausea, malaise,


and a low-grade fever. She has a history of significant alcohol abuse and still continues to
drink. She came to the emergency department with an episode of melena a month ago and
was found to have esophageal varices and anemia. Examination shows right upper
quadrant tenderness and hepatomegaly. Which of the following laboratory results are most
consistent with the patient's symptoms? Aspartate aminotransferase of 120 U/L and
alanine aminotransferase of 60 U/L

A 51-year-old man, with a 20 year history of alcoholism, comes to the emergency


department because of a loss of appetite, abdominal pain, and fever for the past 24 hours.
He says that he had consumed twelve beers and a bottle of vodka two days ago.
Laboratory studies are obtained, and results show the following: Which of the following is
most likely associated with the current presentation? Cytoplasmic inclusion bodies with
keratin

A 65-year-old man with a history of alcoholic cirrhosis and ascites comes to the emergency
department because of seven days of worsening confusion and impaired memory. For the
last two days, he has been increasingly agitated at night and spends a large portion of the
daytime asleep. He denies diarrhea, melena, hematemesis, impaired balance, and double
vision. He takes no medications. His temperature is 38.2°C (100.8°F), pulse is 90/min,
respirations are 16/min, and blood pressure is 130/74 mm Hg. Abdominal examination
shows a tender, distended abdomen with shifting dullness. The rest of the examination and
a computed tomography scan of the head show no other abnormalities. Urinalysis and fecal
occult blood test show no abnormalities. Serum laboratory results are shown below: Which
of the following mechanisms is the most likely cause for this patient's suspected acute
illness? Translocation of bacteria through the intestinal wall

A 50-year-old alcoholic man comes to the emergency department because of an episode of


hematemesis. The patient looks disheveled and is disoriented to time and place. Past
medical history includes hepatitis C infection. Abdominal examination shows abdominal
distension with a fluid wave and caput medusae. Examination of the extremities shows a
bilateral “flapping” tremor, red palms, and bilateral 2+ lower extremity edema. Which of
the following is the most likely cause of the patient's palmar erythema? Hyperestrinism

A 65-year-old man with a history of cirrhosis and hepatocellular carcinoma (HCC) comes to
the emergency department because of 3 weeks of mild but increasing abdominal pain. He
has a history of ascites refractory to salt restriction and diuretic therapy. He endorses
intermittent chills, but no nausea or diarrhea. The patient denies constipation and black
stools, but endorses a notable decrease in how much urine he has produced in the past
week. Physical exam shows the patient’s temperature is 37.8°C (100°F), with otherwise
normal vital signs. Examination of his abdomen shows shifting dullness by percussion. An
abdominal ultrasound shows ascites and no signs of hydronephrosis. Laboratory studies
are shown below: Which of the following is the most likely cause for this patient’s
increased creatinine? Hepatorenal syndrome

A 45-year-old woman comes to the emergency department because of dysfunctional


uterine bleeding. A hysterectomy is performed without complication. The morning after her
hysterectomy, she complains of nausea, headache, and tremulousness. During examination,
she suddenly loses consciousness and experiences a brief tonic-clonic seizure that stops
without intervention. The patient has never experienced symptoms like these before. She
has no history of seizures. Which of the following examination findings is most likely to be
observed in this patient? Cirrhosis

A 52-year-old woman comes to the emergency department because of an episode of


melena. Physical examination shows a distended abdomen with shifting dullness and a
fluid wave. An esophagogastroduodenoscopy shows bleeding from esophageal varices.
The patient is diagnosed with decompensated cirrhosis and placed on the liver transplant
waiting list. Which of the following laboratory value combinations are most appropriate in
determining her priority on the waiting list? Serum bilirubin, serum creatinine, and
international normalized ratio

5: Hepatopatías infecciosas
A 30-year-old woman comes to the clinic because of fever, joint pain, and a rash on her
lower extremities. She admits to intravenous drug use. Examination of her lower extremities
reveals palpable petechiae and purpura. Laboratory testing reveals a negative antinuclear
antibody, positive rheumatoid factor, and positive serum cryoglobulins. Which of the
following underlying conditions in this patient is responsible for these findings? Hepatitis C
infection

A 33 year-old female comes to the office because of increasing fatigue and joint pain for the
past 3 months. She has also noticed a decreased appetite and a 4-kg (8.8-lb) weight loss.
She does not complain of abdominal pain, neurologic symptoms, and she has not noticed
any new rashes. She has always had irregular periods, but her last “regular” period was 8
weeks ago. Her medical history is noncontributory and she takes no medications. She has
been sexually active with her boyfriend for the past 4 months. Her temperature is 37.8°C
(100°F), pulse is 77/min, and blood pressure is 132/88 mm Hg. Examination shows scleral
icterus, spider telangiectasias on the trunk, and mild hepatosplenomegaly. Urine pregnancy
test is negative. Laboratory studies show:* Which of the following is the most appropriate
test to confirm the diagnosis? Liver biopsy

A 26-year-old woman comes to emergency department with right upper quadrant pain and
fever. For the past two weeks, after returning from a trip to India, she has been exhausted
and "itchy all over". On physical exam, she has marked jaundice with tenderness to
palpation in the epigastric region and right upper quadrant. Laboratory results are
remarkable for elevated aminotransferases and bilirubin. If a liver biopsy were performed,
what would the acidophilic bodies seen on biopsy represent? Hepatocyte apoptosis

A 65-year-old man who immigrated from China to the United States 15 years ago comes to
the office because of increased abdominal girth, weight loss, and fever. Physical
examination shows for right upper quadrant tenderness on palpation and organomegaly. A
tissue biopsy is obtained and tests positive for a primary malignancy. Which of the
following tumor markers is most likely to be elevated in this patient? α-fetoprotein (AFP)

A 25-year-old man comes to the emergency department because of fever, jaundice, and
pain localized to the right upper quadrant. Endoscopic retrograde
cholangiopancreatography shows alternating strictures and dilation of the bile ducts with a
characteristic "beading" appearance with the involvement of both intrahepatic and
extrahepatic bile ducts. Which of the following is the most likely diagnosis? Primary
sclerosing cholangitis

A 62-year-old man is admitted to the hospital because he's been having fatigue, joint pain,
abdominal swelling, and bloating for 2 weeks. He has a history of hypertension and was
recently diagnosed with type 2 diabetes mellitus. He hasn't any recent illnesses, fever, or
chills. On exam, his skin is tan and cracked, and he has palmar erythema. He also has
scleral icterus and prominent abdominal distention with a fluid wave. During his hospital
stay, a liver biopsy is performed and Prussian blue stain is applied to the slide (shown
below). Given these results, which of the following is the most appropriate treatment
option? Begin serial phlebotomy

A 15-year-old boy comes to office with complaints of nausea, vomiting, diarrhea and
malaise. He has diffuse abdominal discomfort and has not eaten much in the past few days.
Physical examination reveals a temperature of 39o C (101.8o F). Laboratory findings show
a normal complete blood count, low bicarbonate levels, hypokalemia, hypernatremia and
elevated aminotransferases. He appears pale and fatigued but is not jaundiced. Which of
the following is most likely to reveal the source of his symptoms? Hepatitis A IgG and IgM
serum titers

A 19-year-old woman, gravida 2, para 1, presents to your clinic to follow up on her first
trimester laboratory results. She is now at 20 weeks of gestation and has had an
unremarkable pregnancy thus far. Her laboratory workup was negative for HIV, chlamydia,
gonorrhea, and syphilis. Her hepatitis panel demonstrated the following results: HBsAg:
positive, Anti-HBsAb:negative, Anti-HBcAb:negative. A hepatic function panel drawn in
clinic demonstrates normal AST and ALT values. What is the next best step in the
management of this patient? Obtain HBV DNA, anti-HBeAb, and HBeAg levels

A 46-year-old woman comes to the clinic because of abdominal pain, unintentional weight
loss, and low-grade fever. The abdominal pain is dull and located in the right upper
quadrant, and she has lost 4.35-kg (10-lb) in the last three months. The patient states she
has intermittent yellowing of skin, dark urine, and clay-colored stool. Examination shows
mild hepatomegaly and jaundice. Right upper quadrant ultrasound shows ductal dilatation
and a mass along the lower margin of the liver. Which of the following is the most
appropriate next step in management? Conduct CT abdomen and pelvis with contrast

A 50-year-old man, who is currently homeless, is brought to the emergency department


because he was found unconscious. He appears intoxicated and unable to provide a
detailed history. Physical examination shows that he is febrile with a temperature of 38.3º
(101ºF) and appears malnourished. He has erythematous macules and purpuric papules on
his buttocks and lower extremities. He complains of pain in numerous joints and joint
tenderness upon palpitation. His laboratory workup is as follows: Which of the following is
the most likely diagnosis? Circulating cryoglobulins

A 15-year-old girl comes to the office for her annual visit. Physical examination shows mild
jaundice and scleral icterus. She says that she recently began taking oral contraceptives.
Laboratory studies show conjugated hyperbilirubinemia. Her AST is 15 U/L and her ALT is
18 U/L. Upon oral cholecystography, the biliary system cannot be visualized, even with
double contrast. Which of the following is the most likely diagnosis leading to these
findings? Dubin–Johnson syndrome

A 60-year-old rock musician comes to the office because he has been feeling increasingly
tired for 6 months. He has a history of intravenous drug use and alcohol abuse. He states
that he feels quite tired, but otherwise has no complaints. Examination is noncontributory.
His laboratory investigations are normal aside from elevated liver enzymes. Which of the
following additional tests should you order first? Hepatitis C virus RNA

A 60-year-old man who grew up in rural Australia before moving to the east coast of the
United States last year comes to the office with gradual onset of jaundice. He reports
abdominal pain, particularly in the right upper quadrant area. Physical examination shows
hepatomegaly. Abdominal X-ray shows a single, large calcified cyst in the liver that is
partially obstructing the common bile duct. A PET scan is negative. Based on the clinical
presentation, which of the following treatments is most likely indicated? Aspiration with
ultrasound guidance
A 35-year-old woman comes to the emergency department because of fever, abdominal
pain, diarrhea, and weight loss for the past 3 weeks. She is a recent immigrant from Mexico.
She denies nausea, vomiting, or chills. Physical examination shows severe right upper
quadrant tenderness to palpation and bloody diarrhea. Laboratory analysis reveals
leukocytosis, elevated liver enzymes, and elevated alkaline phosphatase. A CT is obtained
and shows a hypodense avascular liver mass with well-defined margins. Which of the
following is the most likely diagnosis in this patient? Entamoeba histolytica

A 33-year-old woman, gravida 2, para 1, comes to the office to follow up on her first
trimester laboratory results. She is now at 20 weeks of gestation and reports no issues with
her pregnancy. She reports that her morning sickness has largely resolved and that her
mood has been stable. Her laboratory workup was largely unremarkable, except for her
hepatitis panel, which returned the following results: HBsAg: negative, HBeAg: negative,
Anti-HBsAb:negative, Anti-HBcAb:negative. When asked about her sexual history, the
patient reports that she only has sexual intercourse with her husband and has not been
using any form of contraception since becoming pregnant. Which of the following is the
next recommended step in the management of this patient? No intervention is necessary at
this point

6: Vía Biliar
A 38-year-old man comes to the outpatient clinic because of severe right upper quadrant
pain and fever. His temperature is 38.8°C (101.8°F), pulse is 80/min, respirations are 16/min,
and blood pressure is 129/79 mm Hg. His examination reveals a positive Murphy sign.
Blood tests show elevated concentration of leukocytes. Right upper quadrant ultrasound of
the abdomen shows gallbladder wall thickening of 6 mm. While the ultrasound is pressing
against the right upper quadrant, he complains of pain and guards his abdomen. Which of
the following is the most appropriate next step in management? Early cholecystectomy

A 6-week-old infant boy, born in the United States, comes to the clinic because of slow
growth, weight gain, foul-smelling stools, and dark urine. His symptoms began four weeks
ago, and have continued to worsen. Physical examination shows an enlarged spleen and
jaundiced skin. Serum AST, ALT, and total bilirubin concentrations are elevated. Abdominal
ultrasound shows a triangular cord sign. A HIDA scan shows obstruction of flow from the
liver into the gallbladder and small intestine. Which of the following is the most
appropriate next step in management? Hepatoportoenterostomy

A 42-year-old woman comes to the emergency department because of abdominal pain.


The pain began five hours ago, is postprandial, steadily increasing in intensity, and
associated with nausea and vomiting. Her temperature is 37.0°C (98.6°F), pulse is 80/min,
respirations are 16/min, and blood pressure is 125/85 mm Hg. Examination shows an
overweight middle-aged woman in mild distress. She has no signs of jaundice. Her
abdomen is exquisitely tender in the right upper quadrant. Ultrasound shows several
gallstones, an edematous gallbladder wall, and a positive sonographic Murphy sign. For
which of the following malignancies is this patient most likely at an increased risk?
Carcinoma of the gallbladder

A 75-year-old woman comes to the emergency department because of abdominal pain and
distention. The abdominal pain is colicky, and associated with nausea and vomiting. For
three days she has not been able to pass flatus or stool. Her temperature is 37.0oC
(98.6oF), pulse is 99/min, respirations are 15/min, and blood pressure is 125/75 mm Hg.
Abdominal examination shows high-pitched bowel sounds. Abdomen CT is done. Which of
the following is the most likely diagnosis? Gallstone ileus

A 40-year-old woman comes into the emergency department because of increasing fatigue
over the past two weeks. She reports feeling ill a few weeks ago with fever, chills, and
abdominal pain. Her temperature is 37.9°C (100.2°F), pulse is 75/min, respirations are
15/min, and blood pressure is 120/85 mm Hg. Physical exam shows no abnormalities.
Serum aminotransferase concentration is 290 U/L, and serum alkaline phosphatase is 100
U/L. Cholangiography shows multifocal stricturing and dilation of both the intrahepatic and
extrahepatic bile ducts. Which of the following may also be associated? Night blindness

A 49-year-old man comes to the outpatient clinic because of right upper quadrant pain,
nausea, and occasional vomiting. His temperature is 37.0°C (98.6°F), pulse is 76/min,
respirations are 16/min, and blood pressure is 125/75 mm Hg. Examination shows right
upper quadrant tenderness and jaundice. Laboratory tests show elevated bilirubin and
alkaline phosphatase, but WBC is normal. Pancreatic enzyme levels are normal.
Trans-abdominal ultrasound shows a stone blocking and dilating the common bile duct.
Which of the following is the most appropriate next step in management? Perform an
endoscopic retrograde cholangiopancreatography (ERCP)

A 42-year-old woman comes to the emergency department because of postprandial right


upper quadrant pain (RUQ) radiating to the back, nausea, jaundice, and vomiting over the
past 12 hours. Her temperature is 38.8°C (101.8°F), pulse is 85/min, respirations are
15/min, and blood pressure is 125/75 mm Hg. Abdominal examination shows
non-distended abdomen with right upper quadrant tenderness upon palpation. The liver,
rectal, and pelvic examinations are normal. Ultrasound of the right upper quadrant shows
gallbladder wall thickening and biliary dilation. Laboratory studies show: Which of the
following is the most appropriate next step in management? Admit patient to the hospital,
provide intravenous hydration, begin antibiotic therapy, and recommend ERCP
A 72-year-old woman comes to the emergency department because of intermittent
abdominal pain associated with persistent nausea and vomiting for the last five days. She
had an episode of vomiting with hematemesis six hours ago. Her past medical history is
significant for diabetes mellitus and hypertension. Her current medications are metformin
and lisinopril. Her temperature is 38.2°C (100.8°F), pulse is 118/min, respirations are
22/min, and blood pressure is 110/60 mm Hg. Physical examination shows abdominal
distention and tenderness as well as increased bowel sounds. Laboratory studies show: In
addition to signs of partial or complete intestinal obstruction, which of the following is a
radiologic finding that might be present in this patient? Air in the biliary tree on plain
abdominal radiograph.

A 43-year-old woman comes to her primary care physician because of right upper quadrant
pain. The pain occurs only after eating a fatty meal. She denies any sick contacts, fever, and
chills. Her temperature is 37.6°C (99.5°F), pulse is 92/min, respirations are 18/min, and
blood pressure is 125/80 mm Hg. AST and ALT concentrations are within normal limits. The
right upper quadrant ultrasound of her abdomen is inconclusive because of her body
habitus and a follow-up CT scan of the abdomen is ordered, the results of which are shown
below. Which of the following is the most likely diagnosis? Cholelithiasis

A 40-year-old woman comes to the emergency department because of a 12-hour history of


severe abdominal pain in the right upper quadrant. In the past she has experienced
intermittent pain in the same location that lasted up to one hour, but it has always been
transient. She also notes vomiting her meal last night and has had anorexia for one day. Her
temperature is 37.8°C (100.0°F), pulse is 95/min, respirations are 18/min, blood pressure is
120/80 mm Hg, and her oxygen saturation is 97% on room air. She has tenderness and
guarding in the right upper quadrant. Which of the following is the most appropriate next
diagnostic step? Sonography

A 55-year-old man comes to the emergency room because of abdominal pain. The pain
started the previous evening and is similar to abdominal pain he has experienced in the
past. The patient has a history of hypertension, hyperlipidemia, obesity, alcohol use,
osteoarthritis, and peripheral vascular disease. His medications include hydrochlorothiazide,
pravastatin, metformin, and ibuprofen. His temperature is 37°C (98.6°F), pulse is 120/min,
respirations are 18/min, and blood pressure is 135/85 mm Hg. Physical examination shows
epigastric and right upper quadrant tenderness on palpation of his abdomen. Heme-occult
stool test is negative. Serum lipase is normal. Which of the following is the most likely
underlying cause of his condition? Obstruction of the cystic duct

A 56-year-old man comes to the emergency department because of severe abdominal pain
for 3 hours. He states that he has never had anything like this before and that he has
vomited 4 times. He thinks his symptoms may be due to consuming some eggs that were
past their expiry day. His temperature is 36.8°C (98°F), pulse is 87/min, respirations are
18/min, and blood pressure is 117/78 mm Hg. Examination shows that his abdomen is soft
and non-tender. A CT scan is obtained, and a radiologist reports an incidental finding, but
no acute intra-abdominal pathologies. Based on the incidental finding shown, which of the
following malignancies is this patient most likely to develop? Gallbladder adenocarcinoma

A 35-year-old woman comes to the emergency department because of progressively


worsening right upper abdominal pain for two days. The patients states that for the past 3
months she's had a similar pain 1-2 hours after meals, but now the pain is more severe and
constant. She admits to fever, dark-colored urine, and clay-colored stools. She denies pain
radiation, nausea, or vomiting. Physical examination shows a middle aged female in
moderate distress with mild jaundice. Abdominal examination shows tenderness to
palpation in the upper right quadrant. A pregnancy test is negative. A complete blood count
shows leukocytosis and elevated alkaline phosphatase. Blood culture is positive for
Escherichia coli. Based on the patient's presentation, which of the following is the most
likely diagnosis? Ascending cholangitis

A 64-year-old man comes to the outpatient clinic because of abdominal pain. He reports
that for the last few months, he has had postprandial pain that is made worse by spicy
foods. He states that the pain is oftentimes located in the right upper portion of his
abdomen and feels like it's traveling to his shoulder blade. These episodes are sporadic and
unpredictable. He denies any fevers. Physical examination shows no abnormalities.
Abdominal ultrasound is shown below. Which of the following is the best treatment for this
condition? Cholecystectomy

A 35-year-old woman comes to the emergency department because of episodic abdominal


pain. The pain is located in the right upper quadrant, lasts for about 30-minutes per
episode, is usually postprandial, especially after fatty meals, and is sometimes associated
with nausea and vomiting. The patient does not have a family history of
hypertriglyceridemia and does not drink alcohol. Which of the following findings is
expected on abdominal examination and initial laboratory work? Physical examination and
laboratory findings without abnormalities

A 6-week-old infant born in the United States comes to the clinic because of slow growth,
weight gain, foul-smelling stools, and dark urine. His symptoms began four weeks ago and
have continued to worsen. He has an enlarged spleen and is jaundiced. Serum AST, ALT,
and total bilirubin concentrations are elevated. Abdominal ultrasound shows a triangular
cord sign. A HIDA scan shows obstruction of flow from the liver into the gallbladder and
small intestine. Which of the following is the most likely diagnosis? Biliary atresia
A 45-year-old woman comes to the emergency department because of the sudden onset of
abdominal pain during the past hour. She reports occasional episodes of abdominal pain
that radiates to the right shoulder, especially after eating greasy meals; however, her usual
episodes are milder and shorter. She does not drink alcohol or use illicit drugs. Her
temperature is 39.4°C (103°F). Physical examination shows scleral icterus. Which of the
following is the most likely diagnosis? Cholangitis

An 85-year-old woman comes to the emergency department because of colicky abdominal


pain and constipation. She says she has not been able to pass gas or stool for the past two
days, and that her abdomen has expanded in the last few days. Her temperature is 37.0°C
(98.6°F), pulse is 85/min, respirations are 16/min, and blood pressure is 130/75 mm Hg.
Abdominal examination shows high-pitched bowel sounds, distention, but no guarding or
rebound. Her BMI is 45 kg/m2. CT abdomen scan results are obtained and shown below.
Which of the following findings is being highlighted in this image? Rigler triad

A 53-year-old man comes into the emergency department because of a sudden onset of
excruciating epigastric pain radiating to his upper back. The pain was post-prandial and
associated with multiple episodes of emesis without relief. The vomit consists of digested
food and a green mucous-like substance. Fever and acute abdominal pain began as soon as
he stopped vomiting. The patient has familiar hypertriglyceridemia and chronic
cholelithiasis. His temperature is 38.8°C (101.8°F), pulse is 115/min, respirations are
16/min, and blood pressure is 145/85 mm Hg. Examination shows abdominal guarding and
severe upper epigastric tenderness radiating to his upper back. Cullen sign and Grey Turner
sign are absent. Which of the following diagnostic tests is the most appropriate next step in
management? Abdominal ultrasonography

A 43-year-old woman comes to the emergency department because of a 1-day history of


severe nausea and projectile vomiting and right upper quadrant pain. She has had
intermittent pain in the same area for the past two months, but her symptoms have never
been this severe before. Physical examination demonstrates a positive Murphy’s sign.
Ultrasonography of the abdominal region reveals multiple gallstones. A laparoscopic
cholecystectomy is recommended.Which of the following structures is most likely to be
found in the area bounded by the common hepatic duct medially, cystic duct inferiorly, and
lower edge of the liver superiorly? Cystic artery

A 42-year-old woman comes to the emergency department because of recurrent


abdominal pain that has persisted over the past 3 weeks. The pain is dull, postprandial, and
located in right upper quadrant. She denies nausea and vomiting. Her temperature is 37.5°F
(99.5°F), pulse is 75/min, respirations are 16/min, and blood pressure is 125/75 mm Hg.
Examination shows that her abdomen is non-distended with minimal tenderness. The liver,
rectal, and pelvic examinations are normal. Laboratory studies show: Serum lipase and
amylase concentrations are normal. Which of the following anatomical landmarks, if
obstructed, is most likely involved in this patient's disease process? Cystic duct

A 42-year-old woman comes to the emergency department because of abdominal pain,


nausea, and vomiting. The pain is cramp-like, located in the right upper quadrant, and
radiates to her right shoulder. It is postprandial particularly after a fatty meal. She has also
experienced several episodes of non-bloody, non-bilious vomiting. She endorses three
previous episodes of similar pain in the past three months, but states that her prior
episodes lasted less than four hours. Her temperature is 37.0°C (98.6°F), pulse is 85/min,
respirations are 18/min, and blood pressure is 128/80 mm Hg. Abdominal examination
shows right upper quadrant tenderness with palpation, and no guarding or rebound. An
abdominal ultrasound is performed, but it is difficult to visualize the gallbladder due to her
body habitus. Which of the following is the most appropriate next step step in
management? Cholescintigraphy (HIDA scan)

7: Pancreatitis + abdomen agudo + apendicitis


A 25-year-old woman comes to the emergency department because of abdominal pain.
She describes it as intermittent dull, throbbing pain in the left lower quadrant. She
complains of nausea and vomiting in the morning that usually goes away in the middle of
the day. She denies the possibility of being pregnant as she uses oral contraceptive pills.
Her past medical history includes treatment of a sexually transmitted infection with
doxycycline one year ago, and a history of depression, currently being treated with
paroxetine. Serum beta-hCG levels are 2225 mIU/mL. A transvaginal ultrasound scan
(TVUS) is obtained and is negative for an intrauterine gestation. Which of the following
features on history put this patient at an increased risk for the most likely diagnosis? Prior
infection with Chlamydia trachomatis

A 55-year-old man comes to the emergency department because of a 4-week history of


greasy, foul-smelling diarrhea. He has had an alcohol use disorder for several decades and
has been hospitalized several times in the past for a "stomach problem." He denies fever,
hematochezia, or mucus in his stools. Which of the following medication is most
appropriate in the treatment of this patient? Lipase

A 13-year-old boy is brought to the emergency department for evaluation of testicular pain
and swelling. The patient was running when he felt sudden onset groin pain. Since then, he
experienced severe scrotal pain with associated nausea and vomiting. He has had similar
episodes of pain over the past two months that have all self-resolved without intervention.
He has no significant past medical history and has received all age-appropriate
vaccinations. Temperature is 37.0°C (98.6°F), pulse is 94/min, respirations are 21/min, and
blood pressure is 141/80 mmHg. Physical examination demonstrates an uncomfortable
adolescent male in significant pain. Examination demonstrates an edematous and
erythematous right scrotum. Stroking of the right inner thigh does not result in elevation of
the right testis. The pain is worsened with elevation of the scrotum, and the scrotum does
not transilluminate. Which of the following is the best next step in management? Surgical
fixation of the bilateral testicles

A 42-year-old man comes to the emergency department because of 3-day abdominal pain.
The pain occurs intermittently throughout the day, but is not associated with meals or
sleep. He has a history of rheumatoid arthritis, for which he takes multiple non-opioid pain
relief medications per day. He otherwise appears well in spite of his acute pain. His vitals
show no abnormalities and he has epigastric tenderness without distention. Which of the
following diagnostic tests is the most appropriate next step in the management of this
patient? Complete blood count

A 40-year-old man comes to the emergency department because of increasing upper


abdominal pain that began two hours ago. He states the pain has been progressive, with
associated nausea and several episodes of vomiting. Physical examination shows his
temperature is 37.8ºC (100.2˚F), pulse is 105/min, respirations are 18/min, and blood
pressure is 115/75 mmHg. He appears very pale and his abdomen is extremely tender to
palpation. Laboratory studies show: Which of the following is the most likely underlying
cause of this patient's condition? Gallstone disease

A 60-year-old man comes to the emergency department because of fever and severe
right-sided upper and lower abdominal pain for the past 12 hours. He has had no bowel
action for the past 5 days. He was recently diagnosed with lymphoma of the liver and was
discharged several days ago after starting his first cycle of Hyper-CVAD chemotherapy. His
abdomen is severely distended and there rebound and percussion tenderness exists. A
nasogastric tube is inserted, he is started on IV fluids and antibiotics, and a CT scan is
ordered. Which of the following is the most likely diagnosis? Neutropenic enterocolitis

A 35 year-old investment banker with a history of liver injury is undergoing surgery for an
appendectomy. You are his anesthesiologist; which of the agents below is best suited for
the patient’s anesthesia? Sevoflurane
A 35-year-old woman, grava 1, para 0, at 35 weeks' gestation comes to the emergency
department because of painful vaginal bleeding for the past 2 hours. She says the pain
began suddenly and is located in her lower abdomen and lower back. She is also
experiencing contractions every 10 minutes, which have not increased in frequency or
intensity. Her past medical history is significant for hypertension that has been well
managed throughout her pregnancy with labetalol. She has smoked 10 cigarettes a day for
the past 12 years, and has continued to smoke throughout her pregnancy. She does not
drink alcohol, and has not used illicit drugs. Physical examination shows blood in the
vaginal vault and a closed cervix. Which of the following is the most likely diagnosis?
Placental abruption

A 50-year-old woman comes to the office because of nausea, vomiting, and decreased
appetite for the past four days. Her medical history shows systemic lupus erythematosus
(SLE) and fibromyalgia, for which she requires chronic opiate therapy. Current medications
include prednisone, hydroxychloroquine, and a daily multivitamin. She recently was treated
for community-acquired pneumonia with azithromycin. She works as a nurse at a local
hospital and denies use of drugs, alcohol, or tobacco products. Abdominal examination
shows a soft, distended abdomen that is diffusely tender to palpation. Which of the
following is the most likely underlying cause of her complaints? Hypomotility of the
gastrointestinal tract

A 29-year-old woman, gravida 1, para 0, at 19 weeks gestation, comes to the emergency


department because of nausea, vomiting, fever, and abdominal pain for the past two days.
She says she has lost her appetite. Her temperature is 38.8°C (101.8°F), pulse is 92/min,
respirations are 17/min, and blood pressure is 135/80 mm Hg. Physical examination shows
moderate tenderness to palpation in the right lower quadrant and right flank tenderness.
Which of the following is the most appropriate next step in management? Graded
compression ultrasound

A 65-year-old woman comes to the emergency department because of mid-epigastric pain


that radiates to the back. Despite attempts to alleviate the pain, it has persisted for the past
8 hours. She has not had any melena. She has a history of hyperparathyroidism.
Examination shows temperature is 38°C (100.4°F), pulse is 102/min, and blood pressure is
100/70 mm Hg. Her abdomen is mildly distended, tender to palpation, with no guarding.
Complete blood cell count shows no abnormalities. Serum lipase concentration is elevated.
Which of the following is the greatest risk factor for the likely diagnosis? Hypercalcemia

A 20-year-old woman comes to the emergency department with lower abdominal pain for
the past three days. She says the pain is present on both sides of her lower abdomen, and
is sharp in nature. She also reports irregular vaginal bleeding and lower back pain for the
past seven days, before which she had an intrauterine contraceptive device placed.
Examination shows adnexal tenderness, uterine tenderness and a mucopurulent cervical
discharge. Urine dipstick shows no abnormalities. Gram stain from a cervical swab shows
Gram-positive filamentous rods. The organism responsible for causing this infection can be
characterized by the presence of which of the following? Sinus tracts containing sulfur
granules
A 4-year-old boy is brought to the emergency department by his parents because of severe
abdominal pain and constipation for 2 days with no improvement. Prior to this episode his
mother states that he has become increasingly irritable and lethargic over the past 2 weeks
to the point where he has stopped speaking. Review of the patient's medical history shows
that he failed to meet developmental milestones over the past year. When asked about
this, the parents mention that he has been having a hard time adjusting since they moved
into an older home with many of the original fixtures 15 months before. Physical
examination of the patient shows marked lethargy and is otherwise normal. Based on the
patient's symptoms, which of the following is the most appropriate treatment? Dimercaprol

A 62-year-old man with a history of atrial fibrillation and previous myocardial infarction
comes to the emergency department because of abdominal pain, nausea, vomiting, and loss
of appetite for the past five hours. The patient rates the pain an 8 on a 10-point scale. He
says that the pain is constant and “all over” his abdomen, unable to point to a specific
location. He took 500 mg of acetaminophen but did not notice any improvement in his
symptoms. He also says he had an episode of hematochezia prior to coming to the
emergency department. Which of the following is the most appropriate diagnostic step in
the management of this patient? Contrast CT

A 26-year-old man is brought to the emergency department an hour after being stabbed in
the abdomen during a street fight. The patient is hemodynamically stable. Abdominal
examination shows a knife wound in the right lower quadrant, guarding, and rebound
tenderness. A full body computed tomography scan is done and shows the following
image: Which of the following is the most likely diagnosis? Pneumoperitoneum

A 77-year-old woman comes to the emergency department because of acute abdominal


pain. She rates the pain a 10 on a 10-point scale and localizes it to her umbilicus. She also
says she experiences nausea and vomiting. She denies diarrhea or constipation. She has a
past medical history of congestive heart failure. She has never had any surgeries. She has a
10-pack-year history of smoking cigarettes. Her temperature is 37.0ºC (98.6ºF), pulse is
80/min, respirations are 16/min, and blood pressure is 120/90 mm Hg. Physical examination
shows a soft, nontender, nondistended abdomen with no rebound tenderness or guarding.
The lower extremities have 1+ pitting edema.
Which of the following is the most likely pathological process underlying this patient’s
condition? Occlusion of vasculature supplying the bowel

A 23-year-old nulliparous woman comes to the office because of chronic episodic pelvic
pain. The pain begins 2-3 days before menses and continues throughout the menses after
which it subsides. Menarche was at age 14. She has a regular menstrual cycle of 25 days
with 5-6 days of moderate bleeding. Medical history is significant for recurrent migraine
headaches with aura. She is not sexually active. Physical examination shows a fixed
anteverted uterus and nodularity in the posterior cul-de-sac. The physician offers surgical
therapy, which the patient declines at this time. Which of the following is the most
appropriate option to treat this condition medically? Progestin only pills

A 3-year-old boy is brought to the emergency department by his mother for abdominal
pain and irritability. She states that for the past two weeks he has been increasingly
lethargic and has had frequent spells of colicky abdominal pain with nausea and vomiting.
She also relates that he has recently been fixated on eating ice and soil, and has been
behaving strangely. The patient has not had any recent dietary or lifestyle changes, and
recently went with his parents on fishing trip to Mexico. The patient's vital signs are within
normal limits. Physical examination of the patient shows a well developed male with
marked pallor and lethargy. Laboratory studies show a white blood cell count of
4,000/mm3, hemoglobin of 8.2 g/dL, and a platelet count of 400,000/mm3. Which of the
following is most likely to be associated with the patient's clinical presentation? Lead
poisoning

A 12-year-old boy comes to the emergency department with severe right lower quadrant
abdominal pain. Parents note the patient has felt hot for 3 days. His temperature is 38.9°C
(102.0°F). Laboratory studies show leukocytosis. The patient is brought to the operating
room for an appendectomy, and the surgeon finds visible inflammation around the
appendix, terminal ileum, and mesenteric lymph nodes, but the appendix itself is normal. A
week later the patient begins complaining of joint pain, dysuria, headache, and malaise. He
has had bloody diarrhea for the past week. What is the most likely source of the pathogen
causing his illness? Contaminated pork or milk

A 35-year-old woman comes to the emergency department because of right lower


quadrant abdominal pain that started suddenly and has been present for the past 3 hours.
She also reports a small amount of vaginal bleeding. Her last period was 6 weeks ago. She
denies fever, nausea, or vomiting. She reports that she is sexually active and had a tubal
ligation 2 years ago. Her temperature is 37°C (98.6°F), pulse is 77/min, respirations are
12/min, and blood pressure is 110/70 mm Hg. A urine pregnancy test is obtained and is
positive. Serum hCG is 1950 mIU/mL. Which of the following is the most appropriate next
step for diagnosis? Transvaginal ultrasound

A 12-year-old previously healthy boy is brought to the emergency department by his


mother because of nausea, vomiting, fever, non-bloody diarrhea and cramping abdominal
pain for the past 24 hours. His mother states that he woke up the day before with a fever of
38.6°C (101.6°F) and has been having 5-6 loose stools per day ever since. She also
mentions that he was helping her cook chicken cutlets two days ago. Physical examination
shows a dehydrated male in moderate distress, and abdominal examination shows diffuse
tenderness to palpation without rebound. Stool culture grows gram-negative,
lactose-negative rod that form translucent colonies on MacConkey agar. Which of the
following is the best course of treatment? Replace fluids and electrolytes

A 10-year-old girl comes to the outpatient clinic because of bloating and excessive
flatulence. Her symptoms worsen when she eats cheese or yogurt. She has been growing
at average pace and does not show signs of malnutrition. She is afebrile. Physical
examination shows abdominal distention, but no signs of tenderness or guarding. Which of
the following is the most appropriate next step in diagnosis? Hydrogen breath test

A 22-year-old woman comes to the clinic because of several days of worsening abdominal
pain. The patient has had chronic pain for about a year, but it has worsened acutely in the
past three days after contracting the flu. The pain is localized in the epigastric region and
radiates to the back. Her stools are oily and float in the toilet. She has also been thirstier
than usual and has been urinating more frequently. Her fasting blood glucose concentration
is significantly elevated, but aminotransferase and alkaline phosphatase concentrations are
within normal limits. She drinks alcohol only on social occasions. Family history includes a
mother with diabetes and a brother with chronic abdominal pain. The patient is sent for an
abdominal CT scan, the results of which are obtained and shown below. Which of the
following is the most likely cause of this patient's symptoms? Mutation in PRSS-1 gene

A 29-year-old pregnant woman comes to the emergency department because of acute,


severe right upper quadrant abdominal pain. She has been receiving regular prenatal care
and has had no complications in her pregnancy thus far. She denies any tobacco use and
has not consumed any alcohol for the past 11 months. Her hepatojugular reflex is negative.
Abdominal examination shows hepatomegaly with an abdominal fluid wave. Urinalysis
shows no abnormalities. Serum laboratory studies show a creatinine concentration of 1.6
mg/dL and an albumin concentration of 2.5 g/dL. Liver enzyme studies show alkaline
phosphatase concentration of 40 U/L, AST concentration of 159 U/L, and ALT concentration
of 184 U/L. Complete blood count shows a leukocyte count of 8,500/mm3. Which of the
following is the most likely diagnosis? Hepatic vein thrombosis

A 49-year-old man comes to the emergency department because of intense abdominal


pain. The pain is sharp in character and associated with nausea and vomiting. He has type 1
diabetes, which he has managed with insulin since childhood. He has also been
experiencing abdominal pain all day but thought it was secondary to the flu. His
temperature is 38.8°C (101.8°F), pulse is 70/min, respirations are 18/min, and blood
pressure is 125/70 mm Hg. Abdominal examination shows right lower quadrant tenderness
with palpation and a positive psoas sign. Which of the following is the most appropriate
next step in management? Laparoscopic removal of the infected organ

A 20-year-old woman comes to the emergency department because of bilateral lower


abdominal pain for the past two days. She states that she had an intrauterine device placed
about 2 weeks before, and she has been experiencing irregular vaginal bleeding and lower
back pain ever since. Pelvic examination shows adnexal tenderness and a mucopurulent
cervical discharge. Gram stain shows gram-positive filamentous rods, and a pregnancy test
was negative. Which of the following is the most likely microorganism causing the patient's
condition? Actinomyces israelii

A 25-year-old man comes to the emergency department because of excruciating abdominal


pain. The abdominal pain is localized to the right lower quadrant. A day ago, the abdominal
pain was located in the periumbilical region and was less severe. His temperature is 37°C
(98.6°F), pulse is 95/min, respirations are 18/min, and blood pressure is 130/80 mm Hg.
Abdominal examination shows tenderness with deep palpation. Laboratory studies show a
leukocyte count of 15,000/mm3. Which of the following is the most sensitive finding for the
patient's likely diagnosis? Pain 3-5 cm from the anterior superior iliac spine on a straight
line from the anterior superior iliac spine to the umbilicus

A 56-year-old man comes to the emergency department because of abdominal pain for the
past two days. He has a history of alcohol abuse and depression and the pain began a day
after a night of heavy drinking. He states that his pain moves from his belly to his back and
rates it as a 7 on a 10-point scale. He also complains of nausea and vomiting. Current
medications include 20 mg of fluoxetine daily. His temperature is 38.0°C (100.5°F) and
physical examination shows epigastric and right upper quadrant tenderness with voluntary
guarding but no rebound tenderness. Which of the following is the most likely diagnosis?
Acute pancreatitis

A 74-year-old woman arrives via ambulance to the emergency department complaining of


increasing abdominal pain, lethargy, and anorexia beginning three days ago. She reports
some nausea, but denies vomiting. Her last bowel movement was two days ago. Upon
arrival in the emergency department, her vital signs are as follows: T 100.0 F P 75 bpm R
14 bpm BP 100/70 SaO2: 99% (room air). Physical exam shows an ill-appearing elderly
woman with a distended, diffusely tender abdomen that is tympanic to palpation. An
abdominal X-ray is ordered, which shows the following: Which of the following is the best
initial step in the management of this patient? Reduction via flexible sigmoidoscopy

A 66-year-old man is brought to the emergency department because of worsening left


lower quadrant pain and diarrhea over the past 2 days. His temperature is 38.7°C (101.6°F),
pulse is 115/min, respirations are 20/min, and blood pressure is 145/85 mm Hg. Physical
examination shows left lower quadrant tenderness. Laboratory studies show leukocytosis.
Which of the following is the most appropriate next step in diagnosis? Computed
tomography scan

A 56-year-old man comes to the emergency department because of onset of severe


epigastric pain. He has intractable retching without vomiting. Physical examination shows a
distended abdomen that is dull to percussion. An X-ray with barium contrast is ordered and
the results are shown below. Which of the following is the most likely diagnosis?
Organo-axial volvulus

A 51-year-old woman comes to the clinic because of abdominal pain for the past 12
months. The pain has been vague, located in the epigastric region and left upper quadrant
of the abdomen. She says she has weekly nausea and vomiting, and intermittent pain in her
left shoulder. An arterial angiogram is obtained and is shown below. Which of the
following is the most likely diagnosis? Splenic artery aneurysm

A 60-year-old man with a history of coronary artery disease and tobacco use comes to the
emergency department because of acute abdominal pain. He has had diffuse mild
abdominal pain after meals for the past three months. However, recently he has
experienced more frequent episodes of abdominal pain and has seen blood in his stool
intermittently for the past two weeks. He rates the pain a 7 on a 10-point scale. Abdominal
examination shows no abnormalities. A computed tomography scan of the abdomen scan
shows bowel wall edema around the splenic flexure extending to the descending colon.
Decreased blood flow in which of the following arteries would most likely account for the
patient's clinical features? Inferior mesenteric artery

A 44-year-old man who was treated four weeks ago for acute pancreatitis comes to the
office with a recurrent episode of abdominal pain, nausea, a "bloated feeling," and slight
abdominal distention. His history is significant for long-term alcohol abuse. Transabdominal
ultrasound shows walled-off fluid collections in the pancreas which are confirmed by a CT
scan as pancreatic pseudocysts, the largest of which is 4.5cm. Which of the following is the
most appropriate treatment plan for this patient? Endoscopic drainage

A 24-year-old woman presents to the emergency department with nausea, vomiting, and
worsening abdominal pain for the last 12 hours. She states the pain started all of sudden
with no inciting factors. Her symptoms were not associated with any particular meal, and
she denies any recent travel. The patient has a history of mild asthma, for which she takes
an albuterol inhaler as needed. Her temperature is 39°C (102.2°F), pulse is 84/min,
respirations are 18/min, and blood pressure is 128/74 mmHg. Physical examination shows
right lower abdominal pain that is elicited with deep palpation. There is no
hepatosplenomegaly. Cardiac and pulmonary exams are noncontributory. A urine β-hCG
test is ordered and returns negative. Leukocyte count is 12,000/mm3. Which of the
following is the next best step in management? Computed tomography of the abdomen

A 27-year-old woman, gravida 2, para 1, abortus 1, comes to the emergency room because
of right lower quadrant pain and nausea. She has a history of ectopic pregnancy and is
concerned that this could be the problem. Her last menstrual period (LMP) was 6 weeks
ago. A quantitative b-hCG concentration is 1,851 IU. Which of the following imaging
modalities would you use for further investigation? Transvaginal ultrasound

A 22-year-old previously healthy man comes to the emergency department because of


severe epigastric pain that began after returning to his hotel room after a dance party. The
pain is constant, radiates to his back, and he has vomited twice since the onset of his
symptoms. He consumes alcohol on a daily basis and attributes this to his job as a disc
jockey. His temperature is 38.6°C (101.5°F), pulse is 115/min, respirations are 22/min, and
blood pressure is 120/80 mm Hg. His abdomen is tender to palpation over the epigastric
area. Laboratory studies show: Which of the following findings most likely increases the
risk of mortality for this patient? The patient's leukocyte count

A 1-week-old infant is referred to a neonatal consultant because of abdominal distension


and rectal bleeding. The infant was observed in the neonatal intensive care unit after
premature birth at 35 weeks' gestation. Abdominal radiography series shows pneumatosis
intestinalis. Which of the following is the most likely diagnosis? Necrotizing enterocolitis

A 62-year-old woman comes to her outpatient provider seeking evaluation for postprandial
abdominal pain. The symptoms started 6 months ago and have progressively worsened.
She reports that the pain begins 30-40 minutes after meals. She has been eating less due
to the pain, and she has experienced a 5 kg (11 lb) weight loss since symptom onset. The
patient denies nausea, vomiting, and diarrhea. Past medical history is notable for
hypertension and hyperlipidemia. The patient has a 40-pack-year smoking history. Her
younger sister was diagnosed with colorectal cancer at age 54. Laboratory testing reveals
the following findings: Which of the following is the next best step in the management of
this patient? Perform CT angiography

A 22-year-old woman comes to the emergency department with lower abdominal pain for
the past five days. She says the pain is present on both sides of her lower abdomen, and is
sharp in nature. She also reports irregular vaginal bleeding and lower back pain for the past
seven days, before which she had an intrauterine contraceptive device placed. Examination
shows adnexal tenderness, uterine tenderness and a mucopurulent cervical discharge.
Urine dipstick shows no abnormalities. Gram stain from a cervical swab shows
Gram-positive filamentous rods. Which of the following is the most appropriate
pharmacological treatment for this patient? Penicillin G

A 48-year-old woman is reviewed on the surgical ward because of fever and wound
discharge for 30 minutes. She had an open appendectomy for gangrenous appendicitis 5
hours ago and her post-operative recovery has been uneventful so far. Her medications
include metformin and intermediate-acting insulin. Her temperature is 38.0°C (100.4°F),
pulse is 87/min, respirations are 18/min, and blood pressure is 117/78 mm Hg. Physical
examination shows that the area around the bandage has become extremely edematous
and has taken on a dusky grayish-purple color. There is also some thin, watery brown
discharge that is draining from the wound. Which of the following is the most appropriate
next step in management? Surgical debridement

A 56-year-old man comes to the emergency department because of significant abdominal


pain, which started eight hours ago. On general examination, he is in severe distress,
diaphoretic, and nauseated. Physical examination reveals absent bowel sounds and severe
rebound tenderness. An abdominal computed tomography is performed. Which of the
following is the most likely diagnosis? Small bowel infarction with subsequent lumenal air
entering into portal circulation

A 53-year-old-man comes to the office for follow-up after being hospitalized for his first
episode of acute alcohol-induced pancreatitis last month. He feels well and denies any
abdominal pain, nausea, anorexia, or diarrhea. His medical history is significant for heavy
chronic alcohol use, and he does not take any medications. His immediate family members
are all healthy. He smokes one pack of cigarettes per day. His vitals are within normal
limits. Laboratory results show serum electrolytes, cholesterol and triglyceride levels are
within normal limits. In addition to limiting alcohol intake, which of the following is most
likely to reduce this patient’s risk of developing chronic pancreatitis? Smoking cessation

A 64-year-old man comes to the emergency department because of a 2-day history of


uncontrollable nausea, vomiting, and generalized abdominal pain. His temperature is
36.8°C (98.2°F), pulse is 112/min, respirations are 20/min, and blood pressure is 104/64
mm Hg. Physical examination shows a distended abdomen with decreased bowel sounds
that is tympanitic to percussion. An abdominal radiograph is obtained and is shown below.
Which of the following is the most common underlying cause of the patient's symptoms?
Adhesions

A 40-year-old man comes to the emergency department because of increasing upper


abdominal pain which began four days ago. He says the pain is constant, with associated
nausea and several episodes of vomiting. His temperature is 37.9ºC (100.2˚F), pulse is
105/min, respirations are 18/min, and blood pressure is 115/75 mmHg. The patient appears
very pale and his abdomen is extremely tender to palpation. Laboratory studies show:
Which of the following is the most likely diagnosis? Acute pancreatitis

A 26-year-old woman comes to the emergency department because of lower abdominal


pain and vaginal bleeding for the past 24 hours. Her last menstrual period was 7 weeks
ago. She is sexually active with her boyfriend and they use condoms intermittently. Physical
examination reveals mild tenderness in the left lower quadrant and left adnexa. The cervix
is closed with dried blood visible at the os; there is no active bleeding. A quantitative serum
B-HCG is 1,500 IU/L. Transvaginal sonography of the patient's left adnexa reveals an
extra-uterine gestational sac with a diameter of 2 cm. Which of the following is the most
appropriate management option for this patient? Intramuscular methotrexate

A 66-year-old man comes to the emergency department because of severe abdominal pain
in his right lower quadrant. The abdominal pain is sharp in character and has been present
for three hours. His temperature is 37°C (98.6°F), pulse is 95/min, respirations are 18/min,
and blood pressure is 130/80 mm Hg. Abdominal examination shows tenderness with deep
palpation at McBurney point. There are no peritoneal signs. Laboratory studies show a
leukocyte count of 16,000/mm3. Which of the following is the definitive treatment?
Appendectomy

A 35-year-old woman comes to the clinic for a fertility consultation. The patient has been
married to her husband for the past 10 years and they have been trying to have a child for
the past 2 years without success. She has been receiving hCG injections for the past 4
months and she says that she has been experiencing nausea, abdominal bloating, excessive
weight gain, darker urine, dyspnea, and lower abdominal pain for the past 2 weeks. The
patient says that symptoms have been worsening. Which of the following is the most likely
diagnosis? Ovarian hyperstimulation syndrome

A 26-year-old woman, gravida 2, para 2, comes to the office because of episodic right
abdominal pain for the past week. She says the pain is dull and intermittent, located in the
right upper quadrant. Her medical history is significant for well controlled hypertension, an
appendectomy at the age of 14, and two prior cesarean deliveries. Her medications include
cilazapril and the combined oral contraceptive pill. Her vital signs show no abnormalities.
Examination shows moderate tenderness in the right upper quadrant without guarding or
rebound tenderness. A abdominal computed tomography scan shows a 5cm heterogenous,
well-circumscribed lesion in the right lobe of her liver (shown below). Which of the
following is the most likely diagnosis? Hepatic adenoma
A 3-year-old boy is brought to the clinic because of irritability, vomiting, fever, and
abdominal pain for the past month. His temperature is 39.2°C (102.56°F), pulse is 110/min,
respirations are 25/min, and blood pressure is 110/70 mm Hg. Physical examination shows
an upper right quadrant abdominal mass on palpation. A CT scan image is obtained and is
shown below. Which of the following is the most likely diagnosis? Wilms tumor

A 38-year-old woman, with a history of gastric bypass two months ago, comes to the
emergency department because of colicky abdominal pain, severe nausea, and diarrhea.
She has a blood pressure of 86/50 mmHg, her heart rate is 120 bpm, and her skin is
sweaty. Her pupils are somewhat dilated but are equal, round, and reactive to light. She
reports a rapid onset of symptoms following a meal out with friends at a local fast food
restaurant, where she had a milkshake and several cookies. Which of the following
complication of bypass surgery is she experiencing? Dumping syndrome

A 58-year-old man comes to the emergency department because of severe abdominal pain
and vomiting. He has felt this pain for months, but it has become intolerable over the last 8
hours. He has a history of gastroesophageal reflux disease (GERD), hypertension, and
asthma. He has not had a bowel movement or passed flatus in 3 days. Since the pain
worsened, he is now having trouble moving. His temperature is 39.1°C (102.5°F), pulse is
115/min, and blood pressure is 135/75 mm Hg. Physical examination shows a distended
abdomen with high pitched bowel sounds. He appears to be in severe distress, secondary
to pain, and refuses to straighten his legs because it hurts too much. Which of the following
is observed on the barium follow-through due to this patient disease? Dilated small
intestines

A 24-year-old man presents to the emergency department with worsening abdominal pain
that began all of a sudden after eating pizza two nights ago. The patient reports the pain
has progressively worsened and is now 8 out of 10 in severity. He reports no change in
bowel movements during this time, but he has had a decreased appetite and moderate
nausea. Medical history includes lactose intolerance. His temperature is 37.9°C (100.2°F),
pulse is 72/min, respirations are 14/min, and blood pressure is 108/66 mmHg. Abdominal
examination shows right lower quadrant pain with passive right hip extension. Which of the
following will most likely provide definitive treatment for this patient’s condition? Surgical
intervention

A 45-year-old man comes to the emergency department because of severe upper


abdominal pain that began suddenly and has intensified over the past six hours. The
patient says that the pain is constant and scores it as a 9 on a 10-point scale. He complains
of nausea but has not vomited. The patient says that the pain radiates to his back and is
somewhat relieved by sitting up and bending forward. He has a history of gallstones.
Physical examination shows epigastric tenderness and reduced bowel sounds. Which of
the following is the most appropriate next step for this patient? Aggressive fluid
resuscitation, nil orally, and analgesia

A 40-year-old woman with four children comes to the emergency department because of
severe epigastric pain that radiates to her back. She says that she has been having pain
which resolves within an hour in the right upper quadrant after meals over the past few
days. A day ago, she developed this same right upper quadrant pain after eating which did
not resolve and then moved to the epigastric area. The pain is still severe and has not
improved over the past day. She also has nausea and vomiting. Physical examination shows
she is afebrile. Her pulse is 110/min, blood pressure 118/62 mm Hg, and BMI is 32 kg/m2.
She has mid-epigastric tenderness with guarding. Laboratory studies show elevated serum
amylase and serum lipase concentrations. Which of the following is most important part of
her initial management? Intravenous fluid

A 23-year-old previously healthy woman comes to the emergency department because of


right sided lower abdominal pain and vaginal bleeding for the past 24 hours. She denies
nausea, vomiting, fever, anorexia, and diarrhea. Her last menstrual period (LMP) was 7
weeks ago. She is sexually active and has multiple sexual partners, using condoms
inconsistently. She is afebrile, pulse is 80/min, respirations are 14/min, and blood pressure is
115/80 mm Hg. Physical examination shows mild tenderness in the right lower quadrant
and right adnexa, with no rebound tenderness nor guarding. The cervix is closed with scant
blood visible at the os; there is no active bleeding. A quantitative serum ßhCG is 2000 IU/L.
Which of the following is the most likely diagnosis? Ectopic pregnancy

8: GEPI (infecciones, parasitosis, diarrea)


A 25-year-old man comes to the emergency department with a fever, headache, and
watery diarrhea. His abdomen has been hurting since last night. He noticed a small amount
of blood in his most recent bowel movement, 2 hours ago. A stool sample is obtained and is
positive for a lactose-negative, hydrogen sulfide-producing bacteria. Which of the following
mechanisms is most likely responsible for causing the symptoms associated with the
causative bacteria? Direct activation of the immune response resulting in increased
intracellular cAMP

A 42-year-old man and his 12-year-old son come to the office because of unremitting
diarrhea for the past two weeks. They have both been having watery, nonbloody, foul
smelling, greasy diarrhea with abdominal pain and cramps. Two weeks ago, they went on a
father-son camping trip to the mountains where they "roughed it." They ate only by
scavenging for edible plants and spearing fish in the river (which they thoroughly cooked
over a campfire), and drank the mountain stream water. A stool sample from each of them
shows the organism below. Which of the following pathogens is the most likely cause of
this diarrhea? Giardia lamblia

A 45-year-old man comes to the emergency department because of severe abdominal pain
and copious non-bloody diarrhea for the last 24 hours. Two weeks ago he was hospitalized
for 7 days to treat acute pancreatitis, and was given antibiotics during his stay. Several days
after being discharged he started feeling more tired and subsequently developed a fever.
He has not traveled recently and has not had any sick contacts since leaving the hospital.
Physical examination shows a male in moderate distress who appears significantly
dehydrated. Abdominal examination shows lower abdominal tenderness to palpation.
Which of the following is a finding associated with the most likely cause of this patient's
diarrhea? Pseudomembranous colitis

A 20-year-old woman comes to the emergency department because of abdominal pain,


fever, and bloody diarrhea for 3 days. She recently immigrated to the United states from
Mexico. She denies nausea, vomiting, or chills. Physical examination shows hepatomegaly
and right upper quadrant tenderness. An indirect hemagglutination titer confirms the
presence of antibodies and an ultrasound shows that she has an abscess in the right lobe of
her liver. Which of the following organisms is most likely responsible for this abscess?
Entamoeba histolytica

A 2-year old girl comes to the clinic because of vomiting and diarrhea for 2 days. Her
mother describes the diarrhea as non-bloody and without mucus. The patient appears to be
in pain and is lethargic. Which of the following organisms is the most likely culprit for the
symptoms in this patient? Norovirus

A 55-year-old man, who recently moved to the United States from Latin America, comes to
clinic because of a 1-week history of bloody diarrhea. Blood tests reveal leukocytosis
without eosinophilia, and liver function tests show elevated alkaline phosphatase and
transaminases. Physical examination of the abdomen shows fever, right upper quadrant
pain, and a palpable mass in the abdomen over the ascending colon. On ultrasound of the
liver, there is a round rim-enhancing lesion which looks like a well-defined abscess with
low density. Chest X-ray shows an elevated right hemidiaphragm. Which of the following
treatments would be most appropriate? Immediately begin treatment with metronidazole

A 14-year-old boy who recently immigrated from Cambodia presents to the clinic with
waxing and waning gastrointestinal symptoms of diarrhea, nausea, and vomiting. On
physical exam, the patient has a mild dry cough with some wheezing, though the heart has
regular rate and rhythm. There are serpiginous, raised erythamatous tracks in the perianal
area of the patient. A stool ova and parasite test is done, the results of which are shown
below. Which of the following is the best treatment for this patient? Ivermectin

A 6-year-old boy comes to the emergency department because of 2 days of diarrhea. His
mother says that her son has also been vomiting occasionally (non-bloody, non-bilious) for
about the same amount of time. He has a heart rate of 120 bpm, a temp of 37.7*C, and a
respiratory rate of 16 breaths/minute. The child appears uncomfortable. His capillary refill is
4 seconds and his mucous membranes are tacky. His abdomen is neither tender nor
distended. The remainder of the exam is unremarkable, and a complete blood count and
electrolyte panel are normal. Which of the following is the most appropriate first
therapeutic step in the management of this child? Oral rehydration therapy

A 4-year-old girl is brought to her pediatrician because of abdominal cramps and diarrhea
for the past two days. The patient's mother noticed that her stools are bloody, contain
mucous and are small in volume. She also mentions that 2 other children at her daughter's
daycare have similar symptoms. Physical examination shows a young girl in moderate
distress with a fever of 39.6°C (103.3°F). Analysis of the patient's stool sample shows
polymorphonuclear leukocytes on a methylene blue stain. A complete blood count was
obtained and shows 14/mm3 white blood cells. Which of the following is the most
appropriate treatment for this patient's condition? Azithromycin

A 20-year-old woman comes to the emergency department with a fever, headache, and
watery diarrhea. The patient's stomach has been hurting since last night and she has
noticed a small amount of blood in her most recent bowel movement. She went on a picnic
with her boyfriend two days ago where they ate peanut butter sandwiches, chips, and had
a couple of soft drinks. A stool sample is obtained and is positive for a lactose-negative,
hydrogen sulfide-producing bacteria. Which of the following is the most likely infectious
agent causing these symptoms? Salmonella enteritidis

A 25-year-old woman comes to the emergency department because of a 2-day history of


nausea, vomiting, subjective fever, profuse diarrhea, and abdominal cramping. She states
that she had attended a large family picnic several days earlier where there were several
cold salads. Following admission, which of the following is the most appropriate first step
in management for this patient? Replacement of fluids and electrolytes

A 4-year-old girl is brought to the office because of diarrhea for 12 hours. Her mother
states that she came home from daycare and developed fever, abdominal pain, and diarrhea
containing flecks of bright-red blood and pus. The girl did not ingest any food at daycare,
and her diet has been unchanged in the last week. A stool specimen is collected and sent to
the lab. Which of the following is the most likely causative organism based on clinical
findings? Shigella dysenteriae

A 22-year-old female comes to the emergency department because of fever, headache and
watery diarrhea for 12 hours. She says her stomach has been hurting since last night, and
that she noticed a small amount of blood in her most recent bowel movement. She states
she went on a picnic with her boyfriend 2 days ago. They ate sandwiches, chips, and
consumed soft drinks. A stool sample is obtained and cultured using MacConkey agar. The
colonies show non-lactose fermenting, non-hydrogen sulfide-producing bacteria. Which of
the following organisms is the most likely to cause such clinical symptoms? Shigella
dysenteriae

A 35-year-old man comes to the emergency department because of sudden explosive


diarrhea and intermittent abdominal pain. His medical history is relevant for recent traveling
to Honduras. Upon further questioning, the patient mentions that he feels an uncontrollable
urgency to defecate that does not cease after defecation. When the patient is asked about
his stools, he describes them as "bloody and foul-smelling." Physical examination shows
decreased alertness, sunken eyes, dry mouth, split skin on mouth corners, and abdominal
tenderness on palpation. His temperature is 38.6°C (101.48°F), pulse is 122/min,
respirations are 24/min, blood pressure is 90/70 mmHg. Laboratory studies show
leukocytosis and elevated alkaline phosphatase. Microscopy of a stool sample is obtained
and shown below. Which of the following is the most appropriate initial treatment for this
patient? Oral rehydration therapy

A 16-year-old African American boy comes to the outpatient clinic because of watery,
non-bloody diarrhea for a month. His diarrhea is associated with abdominal cramps,
bloating, and excessive flatulence, but is not associated with fever or vomiting. He denies
any changes in weight, recent travel history, sick contacts, or recent viral infections. He says
that he has been trying to "bulk up" for football tryouts and has been drinking a lot of
protein powder milkshakes in the past month. Which of the following is the most
appropriate next step in management? Lactose breath hydrogen test

A 30-year-old man comes to the clinic because he plans on traveling to Mexico for three
weeks. He inquires about prophylaxis for traveler diarrhea. Past medical history reveals
inflammatory bowel disease that is well controlled with diet and medication. Which is the
most appropriate prophylactic pharmacological management in this patient? Ciprofloxacin

A 16-year-old man comes to the clinic because of 7 days of flatulence and greasy,
foul-smelling diarrhea. He states he recently went camping and reports that his friend
collected water from a stream, but did not boil or chemically treat the water before they
consumed it. The patient also reports symptoms of nausea, weight loss, and abdominal
cramps followed by sudden diarrhea. Which of the following is the most appropriate
treatment option? Metronidazole

A 4-year-old girl comes to the urgent care clinic because of vomiting and diarrhea for the
past 2 days. Her mother says that many of the other children at her daughter's day care
facility have also been experiencing the same symptoms. She is concerned because some of
the other children have experienced a week or more of diarrhea. On examination, this
patient is febrile to 38C with a heart rate of 150 bpm. Blood pressure is 90/60. Her mucus
membranes are somewhat dry, and abdominal exam reveals hyperactive bowel sounds. In
the interest of ensuring her hemodynamic stability, she is admitted to the hospital and a
sample of her stool is sent for analysis. Bacterial cultures are negative, stool ova and
parasites are both negative, and viral polymerase chain reaction is positive for adenovirus.
Which of the following is the next best step in the treatment of this patient? Continued
support of hydration status

A 3-year-old girl comes to the clinic because of an 8 day history of fever, vomiting and
watery diarrhea. There has been mild improvement in the last few weeks as the frequency
of the patient’s stool has decreased in the last few days, and the stool is more formed. Her
mother denies the patient has had any seizures and reports the child has had a few sick
contacts. The patient goes to a day care about 4 days a week, and three other children have
had similar symptoms. On physical examination the patient has normal vital signs with a
moderate amount of stool in her diaper. She has an otherwise normal exam with no
concerns regarding hydration. Fluids are prescribed to the patient. Which of the following is
the single best treatment option available to help prevent further breakout of this patient's
most likely condition? Return back to daycare with stool well contained in diaper

A 45-year-old man presents to the emergency department due to a several month history
of intermittent diarrhea that has been worsening over the past week. He has about 10
watery stools per day. He also has mild abdominal pain, myalgia, intermittent flushing, and
diaphoresis. Physical examination shows a mildly extended but non-tender abdomen, pulse
is 110/min, dry mucous membranes and a capillary refill of 6 seconds. Laboratory analysis
indicate volume depletion with hypokalemia and a non-anion gap acidosis with a gastric pH
of 4. A tumor producing which of the following endocrine hormones is most likely
responsible for these signs and symptoms? Vasoactive Intestinal Peptide (VIP)

A 30-year-old man comes to the emergency department because of an abrupt onset of


abdominal cramps, diarrhea, nausea, and vomiting that began last evening. He reports
returning from a trip to Mexico yesterday and feeling fine throughout the travels. The
patient feels an urgent need to defecate. Vitals signs are significant for a temperature of
98.0°F (36.7°C). Which of the following is the most likely diagnosis in this patient?
Enterotoxigenic Escherichia coli

A 83-year-old woman comes to the emergency department because of fever, nausea,


vomiting, and profuse watery diarrhea for 2 days. Medical history includes treatment with
antibiotics for a hospital acquired pneumonia 8 days ago. Her temperature is 37.8°C
(100.0°F), pulse is 102/min, respirations are 20/min, and blood pressure is 133/85 mm Hg.
Physical examination shows a distressed appearing woman. Cardiac auscultation is
noncontributory, and pulmonary auscultation shows reveals vesicular sounds. Abdominal
examination shows mild, diffuse abdominal tenderness. Laboratory investigations show the
following: A CT-scan of the abdomen is obtained and shows thickening of the colonic wall.
Which of the following classes of antibiotics was she most likely treated with for her
pneumonia? Fluoroquinolones

A 75-year-old woman comes to the emergency department with worsening shortness of


the breath for the past week. She is admitted with a diagnosis of pneumonia. Her condition
is managed with observation and intravenous levofloxacin 750 mg daily for 2 weeks. On
the 15th day of hospitalization, she has three episodes of diarrhea. The following day, her
white blood cell count is 21,000/mm3, and she begins complaining of abdominal pain. Her
pulse is 110/min, respirations are 18/min, and blood pressure is 92/50 mm Hg. Physical
examination shows she is confused and has diffuse abdominal tenderness without
guarding, rebound, or rigidity. Which of the following is the most appropriate response?
Exploratory laparotomy

A 15-year-old boy comes to the emergency department because of copious amounts of


watery diarrhea for the last 5 days. The patient states the diarrhea began on the last day of
his mission trip to rural Guatemala, where he was helping build a new school with a group
of people from his church. He describes the diarrhea as watery and oily-appearing at times.
He has also been feeling gassy throughout the last few days. Stool microscopy shows the
organisms seen in the image below. Which of the following is the most appropriate
treatment for this patient's condition? Metronidazole

A 12-month-old infant boy comes to the pediatrician's office for a routine well-child
examination. The boy did not have any complications during pregnancy or delivery. He
passed meconium right after delivery. His weight and height are growing at a normal pace;
however, his mother is concerned that he has been irritable and frequently passes gas after
meals. His stool is found to be highly acidic, and he is started on a special diet. Which of the
following is the most likely diagnosis? Lactase deficiency
A 14-year-old girl comes to the office because of fatigue, anxiety, and balding for a month.
Her parents are missionaries and note that her appetite has been odd roughly since they
returned to the United States from a trip abroad. Among her odd behaviors, she has a new
found craving for all things made of ice and the other day they say they caught her eating
dirt. When questioned about her behavior the girl expresses shame but states that she
does these things "almost compulsively." Examination shows a non-tender abdomen with
areas of diffuse fullness, and palpebral pallor. Peripheral smear shows anisocytic,
microcytic, hypochromic red blood cells. Eosinophilia and lymphocytosis are also shown.
Iron studies show decreased transferrin, and decreased ferritin levels. A trichobezoar is
retrieved on upper endoscopy. Which of the following is the most appropriate next step in
treatment? Ivermectin

9: Enfermedad inflamatoria intestinal + malabsorción


A 20-year-old woman comes to the office because of a new rash that appeared two days
ago on her legs as shown below. The lesions are non-pruritic and tender to touch. She was
recently diagnosed with an inflammatory bowel disease that is still being worked-up.
Which of the following is the most likely diagnosis for her dermatological condition?
Erythema nodosum

A 28-year-old man is brought to the emergency department with lethargy and severe
abdominal pain since yesterday. He has had 4-5 episodes of small-volume bloody diarrhea
per day accompanied by lower abdominal cramps for the past 2 months. Review of systems
reveals a 4-kg (8.8-lbs) weight loss and loss of appetite. The remainder of his history is
noncontributory. Temperature is 38.8°C (102°F), pulse is 125/min, respirations are 18/min,
and blood pressure is 90/55 mmHg. Physical examination shows lethargy and dry mucous
membranes. There is abdominal distension, diffuse abdominal tenderness, and decreased
bowel sounds without rebound or guarding. Rectal examination reveals marked tenderness,
with guaiac-positive, maroon-colored, liquid stools in the vault. Laboratory results are
shown below: Fluid resuscitation with 0.9% normal saline improves the blood pressure to
103/59 mmHg. Which of the following is the best next step in management of this patient?
Plain abdominal x-ray

A 15-year-old boy comes to the office because of an intensely itchy rash on his elbows and
knees. He states that the rash began as small vesicles a week ago. Family history is
significant for Graves disease in his mother. Physical examination shows excoriated papules
and erosions that are crusted over on his elbows, knees, and buttocks, as shown below.
Which of the following is most likely to confirm the diagnosis? Skin biopsy

A 37-year-old woman comes to the office because of multiple episodes of diarrhea and
bloating for several years. She thinks she has lost weight because of this indigestion. She
also has some itchy vesicles on her arms and legs. Physical examination shows abdominal
distention as well as mucosal and conjunctival pallor. Antibodies against which of the
following are more likely associated with her condition? Endomysium

A 22-year-old Jewish woman comes to the emergency department because of severe right
lower quadrant abdominal pain for the past two hours. She has had diarrhea and weight
loss for several weeks. A colonoscopy is performed, and an image from the procedure is
shown below. Which of the following is the most likely complication of this patient's
diagnosis? Perianal fistulas

A 29-year-old man comes to the office with complaints of frequent loose stools and
abdominal pain. He started having these symptoms a month ago, and they did not respond
to over-the-counter antidiarrheals. He became concerned when he noticed an increase in
urgency and a small amount of blood admixed with mucoid stools yesterday. Past medical
history is noncontributory. He has not recently travelled outside the country. Vitals are
within normal limits. Physical examination shows mild abdominal tenderness in the left
lower quadrant. Colonoscopy shows erythematous rectal mucosa that is friable to touch,
along with scattered ulcerations extending up to the distal part of sigmoid colon. The
remainder of the colon and ileum are normal. Which of the following is the most
appropriate pharmacotherapy for the patient at this stage? 5-aminosalicylic acid (5-ASA)
suppository

A 40-year-old woman comes to the emergency department because of sudden onset


hemoptysis. She has started a new treatment for her Crohn disease and says that she has
had a low-grade fever and night sweats for the past five days. Which of the following
agents added to her treatment regimen is most likely responsible for her new symptoms?
Infliximab

A 29-year-old man comes to the office because of pain during urination. He was treated for
a urinary tract infection last month. He also says he has progressively worsening diarrhea
over the past two years. These episodes of diarrhea typically last 1-2 weeks, then subside
for several weeks to months. Physical examination shows joint pain in many locations,
especially in his sacroiliac joint. A urine sample is collected and shows pneumaturia. Which
of the following best explains the patient's recurrent urinary tract symptoms? Enterovesical
fistulization

A 28-year-old man of Jewish descent comes to the emergency department because of


abdominal pain, diarrhea, and fever. Physical examination shows a tender abdomen and a
perianal fistula. A biopsy is done and shows chronic inflammation in the small intestine
involving all layers of the intestinal wall. A monoclonal antibody against which of the
following cytokines would be most beneficial in the treatment of this patient? TNFα

A 3-year-old girl comes to the office because her parents say that she frequently has
foul-smelling diarrhea, particularly after eating pasta or bread. Physical examination shows
excoriated papules and vesicles on her elbows, knees, back, and buttocks. She is in the 10th
percentile on the growth curve for height and weight. An endoscopic small bowel biopsy is
obtained and is shown below. Which of the following gastrointestinal neoplasm is she at
the greatest risk of developing if her disease becomes refractory to treatment? T-cell
lymphoma

A 43-year-old man with a recurrent fistula-in-ano comes to the office for monitoring of his
Crohn disease. An MRI is done to determine whether he has developed a complex,
multi-tract fistula. The MRI shows that his fistula only follows one tract. It courses
perpendicularly to the skin from its external perianal opening, then takes a sharp 90° turn
and burrows through both the external anal sphincter and the internal sphincter to open
into the anal canal. According to the Parks classification of fistulas, which of the following
most accurately describes this patient's fistula-in-ano? Trans-sphincteric fistula

A 25-year-old woman comes to the office because of painful new lesions on both of her
legs. The lesions are purple, firm nodules, and her legs are mildly swollen, as seen in the
image below. She has had loose stools, intermittent fevers, and weight loss for the past
three weeks. Which of the following is the most likely diagnosis? Crohn disease

A 29-year-old man comes to the emergency department because of right lower abdominal
pain, bloody diarrhea, and fever for the past week. He says that he has been having these
episodes of right lower abdominal pain and loose stools over the course of the past year.
However, this is the first time he has noticed blood in his diarrhea. Physical examination
shows tenderness along the sacroiliac joint. There are raised erythematous lesions along
his back that are painful to touch. A barium swallow is obtained and the results of which
are shown below: A colonoscopy is performed and shows diffuse granulomatous
inflammation of ileocecal junction with transmural ulcerations. Biopsies show
non-caseating granulomas and skip lesions, with ulcers that are transmural. Which of the
following is most likely to result as a complication of this patient's disease? Fat and vitamin
malabsorption

A 30-year-old woman comes to the office because of increasing fatigue over the past
several months. She was diagnosed with Crohn disease as a teenager, for which she had an
ileocolic resection 3 years ago. She has not had any recent diarrhea or abdominal pain and
says that she has not seen any blood in her stool. She eats a well-balanced diet and takes a
multivitamin regularly. Her fecal occult blood test is negative. Which of the following best
explains this patient's fatigue? Vitamin B12 deficiency

A 40-year-old man comes to the office for a follow-up evaluation of his inflammatory
bowel disease. He asks what the risk factors are, and would like to know what
modifications he can make to his lifestyle that will decrease the risk of exacerbation. Which
of the following is the most appropriate physician response to his question? Smoking may
aggravate Crohn disease and relieve ulcerative colitis

A 15-year-old girl comes to the office because of abdominal pain, diarrhea, flatulence, and
borborygmi. The only known relieving factor is not eating, and as a consequence, she has
lost 6.8-kg (15-lb) in 6 weeks. She is earning excellent grades in school and is involved in
numerous extracurricular activities. She denies any new stress or unusual behavior. She has
noticed numerous fluid-filled blisters along her arms and legs, as shown below. Her
temperature is 37.6°C (99.7°F), pulse is 88/min, respirations 13/min, and blood pressure
118/80 mm Hg. Serologic evaluation is positive for anti-tissue transglutaminase IgA and
anti-endomysial IgA. Which of the following is most likely seen on abdominal radiograph
with barium contrast? Decreased jejunal folds, increased ileal folds

A 38-year-old man comes to the office because of recent onset of abdominal pain around
his umbilicus. He has a history of Crohn disease that was diagnosed a few years ago. He
says that his pants have been feeling a little loose, and he is worried he might have lost
some weight unintentionally. His temperature is 38°C (100.4°F). Physical examination
shows a palpable abdominal mass in the right lower quadrant. Which of the following is
the most appropriate next step in the management of this patient? Abdominal CT

A 29-year-old man comes to the emergency department because of right lower quadrant
abdominal pain, diarrhea, and fever. He says that these episodes have progressively
worsened over the past two years. They typically last 1-2 weeks, then subside for many
weeks to months. During the episodes, he sometimes has bloody diarrhea. Physical
examination shows joint pain in many locations, especially in his sacroiliac joint. There are
multiple erythema nosodum lesions on his back. A biopsy from colonoscopy shows
noncaseating granulomas and skip lesions of transmural ulcers. A barium swallow is done
and the results are shown below. Which of the following is the most appropriate
medication to be administered? Mesalamine (5-ASA)

A 37-year-old woman comes to the office because of multiple episodes of diarrhea and
bloating for several years. She also reports some itchy vesicles on her skin around her arms
and legs. Physical examination shows abdominal distention, but the rest of the exam is
normal except for mucosal and conjunctival pallor. Which of the following is the most
appropriate first-line treatment for her condition? Dietary changes

A 3-year old girl is brought to the office because of a protuberant abdomen and wasted
extremities. Her mother says that her daughter has had recurrent watery diarrhea, poor
appetite, and multiple episodes of emesis. She also reports new-onset bedwetting and
increased drinking behaviors. At her 1-year evaluation, it was noted that she was failing to
thrive. The patient was diagnosed with celiac disease following a duodenal biopsy. There
are no other significant physical findings on the exam. Which of the following tests should
be conducted? Glucose concentrations

A 29-year-old man comes to the emergency department because of right lower abdominal
pain, diarrhea, and fever. He says that he has had episodes of this condition for the past two
years. The episodes typically last 1-2 weeks, then subside for many weeks to months.
During the episodes, he sometimes has bloody diarrhea. Physical examination shows joint
pain in many locations, especially in his sacroiliac joint. There are multiple erythema
nodosum lesions on his anterior shins. A barium swallow is performed, and the result is
shown below. Which of the following best describes the clinical finding shown below?
String sign of Kantor

A 29-year-old man comes to the emergency department because of right lower abdominal
pain and distension, diarrhea, and fever. He says that he has been experiencing these
episodes for two years due to his inflammatory bowel disease. The episodes typically last
1-2 weeks and then subside. His condition is currently being managed by a
gastroenterologist. Despite medical management with 2 medications, his symptoms have
worsened. A biopsy from colonoscopy at the time of diagnosis showed the presence of
noncaseating granulomas and skip lesions with transmural ulcers. A barium swallow is
obtained and shown below. Which of the following treatments is the most appropriate next
step for management of this patient's condition? Surgical resection

10: Síndrome de intestino irritable


A 22-year-old male comes to the office because of constipation for the past 7 months. He
says he has crampy lower abdominal pain that improves upon defecation. His baseline is
defecating twice per week. He has sporadic episodes of multiple stools per day that last
2-3 days. He denies nausea, vomiting, or bloody stools. The abdomen is soft and
nontender; bowel sounds are normal. A colonoscopy and intestinal biopsy are performed.
Which of the following would most likely be seen on light microscopy?
Normal intestinal mucosa
A 15-year-old girl is brought to the office because she’s been having intermittent diarrhea
with associated abdominal pain for the past year. Occasionally the abdominal pain
improves after she has a bowel movement. She experiences frequent stomach aches that
occur without warning, and has tried to avoid various foods including milk, meats, grains,
and caffeine, with only moderate and inconsistent improvements in symptoms. She was
diagnosed with obsessive compulsive disorder three years ago which is now managed with
100 mg of sertraline daily. She denies vomiting, fever, hematochezia, and melena. There is
no family history of gastrointestinal disorders. She is currently in the 60th percentile for
weight given her height and her age. Physical examination shows no abnormalities. A
barium contrast enema shows no abnormalities. Which of the following is the most likely
diagnosis?
Irritable bowel syndrome

A 24-year-old woman comes to the office because of intermittent diarrhea and crampy
abdominal pain over the past year. During these episodes, she notices an increased
frequency of bowel movements and softer stool consistency. Her symptoms get worse with
stress and improve when she takes loperamide. The abdominal pain is decreased after
defecation. She denies bleeding, weight loss, early satiety, family history of colon cancer,
and smoking. Esophagogastroduodenoscopy and colonoscopy with biopsy are
unremarkable. Which of the following is the most likely diagnosis?
Irritable bowel syndrome

A 15-year-old girl comes to the office because of abdominal pain and diarrhea for the past
three months. Her symptoms occur in the mornings before school. Physical examination
shows positive bowel sounds, no abdominal tenderness, and no mass on palpation. Stool
samples, blood work, and colonoscopy and upper endoscopy all show no abnormalities.
Which is the most likely diagnosis?
Irritable bowel syndrome

11: Cancer colorectal


A 40-year-old man comes to the office because of multiple lesions on his forehead. He
states that he otherwise has no current medical issues. His medical history is also
noncontributory. Family history includes a father with small bowel cancer at age 48, and a
sister aged 41 with endometrial cancer. Examination of his forehead is shown below. His
lesions are biopsied and are diagnosed as sebaceous adenomas. Which of the following is
an appropriate next step in the management of this patient? Order a colonoscopy

A 55-year-old man comes to the office because of persistent fevers, fatigue, loss of
appetite, and transient chest pain for 2 months. He also mentions that he has
unintentionally lost 7 kg (15 lb) over the same time period. His temperature is 38.8°C
(102°F), pulse is 87/min, respirations are 18/min, and blood pressure is 110/78 mm Hg.
Examination is noncontributory. Serial blood cultures grow Streptococcus bovis. Once the
patient is stabilized and treated for his bacteremia, which of the is the most appropriate
next step? Colonoscopy

A 61-year-old man comes to his physician for evaluation of excessive fatigue and shortness
of breath. He has also unintentionally lost 12-lb (5.4 kg) in the past 2 months. He has not
had abdominal pain, melena, or changes in bowel habits. He has not visited a physician in
many years, but he was urged by his partner to make an appointment. The patient recently
retired from a battery-manufacturing factory, which he worked at for 35 years. The patient
has smoked a pack of cigarettes daily for the past 40 years. His temperature is 37.0°C
(98.6°F), pulse is 98/min, respirations are 14/min, and blood pressure is 135/85 mmHg.
Physical examination is notable for skin and conjunctival pallor. Laboratory results are as
follows: Hemoglobin 9.8 g/dL Hematocrit 40% Mean corpuscular volume (MCV) 74 μm3
Which of the following diagnostic studies would be most helpful in confirming this patient’s
underlying diagnosis? Colonoscopy

A 50-year-old man comes to the office because of an interest in treatment options for his
recent cancer diagnosis. He has a history of well-controlled hypertension, psoriasis, and
osteoarthritis. He has rectal carcinoma, which is approximately 3.0 cm from his anal verge
with no evidence of metastatic disease. Which of the following is the most appropriate
treatment of choice for this mass? Abdominal perineal resection with permanent colostomy
A 75-year-old African American male comes to the gastroenterology department for his
scheduled screening colonoscopy. His medical history includes well-controlled
hypertension, type 2 diabetes mellitus, and long-term cigarette smoking. Family history
includes endometrial cancer in his mother. His endoscopic examination shows a 2 cm (0.79
in), sessile polyp approximately 6 cm (2.36 in) above the dentate line not amenable to
endoscopic polypectomy. Which of the following is the most appropriate next step?
Multiple endoscopic biopsies of the polyp at the time of initial colonoscopy

A 54-year-old man comes to the office five days after his first screening colonoscopy. He
has no acute complaints. He has a history of hypertension and diabetic nephropathy, both
treated with quinapril. His temperature is 37ºC (98.6ºF), pulse is 89/min, respirations are
14/min, and blood pressure is 130/85 mm Hg. The colonoscopy shows two 0.5 cm polypoid
lesions in the descending colon, both removed by polypectomy. Biopsy findings of one of
the polyps is shown below. Based on the findings above, which of the following best
describes when the patient should have a new colonoscopy? 3 years

A 55-year-old man comes to the office because of persistent fevers, fatigue, loss of
appetite, and transient chest pain for the past 2 months. He has also unintentionally lost 7
kg (15 lb) over the same time period. His temperature is 38.8°C (102°F), pulse is 87/min,
respirations are 18/min, and blood pressure is 110/78 mm Hg. Physical examination is
non-contributory. Serial blood cultures grow Streptococcus gallolyticus. Echocardiogram
reveals vegetations on the mitral valve. Once the patient is stabilized and treated for his
bacteremia, which of the following additional diagnostic tests should be performed?
Colonoscopy

A 60-year-old woman is scheduled to undergo a routine screening colonoscopy for


colorectal cancer. She has no changes in bowel habits, unintentional weight loss, or
noticeable blood in the stool. Her previous colonoscopy 10 years ago showed no
abnormalities. Past medical history is significant for type II diabetes mellitus. There is no
family history of colorectal cancers. Colonoscopy is performed, which reveals a single,
sessile 10-mm polyp covered in mucous in the ascending colon. Histology of this polyp
subtype from a different patient is shown below: Which of the following is the next best
step in the management of this patient? Complete excision of the polyp

You might also like